Med Rev

You might also like

Download as docx, pdf, or txt
Download as docx, pdf, or txt
You are on page 1of 33

Biochemistry: malabsorption, induction of P450 which degrades active vitamin D precursors.

Symptoms: rickets (kids; bending bones), osteomalacia (adult; soft bones),


Fat Soluble Vitamin: vitamins A,D,E,K; precursor for coenzymes. hypocalcemic tetany.
o Requires pancreatic enzymes for absorption (ileum): malabsorption syndrome can o Excess: supplementation (>10 x RDA), sarcoidosis (↑ activation of vitamin D by
cause fat-soluble vitamin deficiency: steatorrhea, cystic fibrosis, sprue epithelioid macrophages). Symptoms: polyuria, polydipsia, nocturia,
o Stored it fat making toxicity possible (unlike water soluble vitamins) hypercalcemia/hypercalciuria (high blood Ca concentration promotes
metastatic calcification), loss of appetite, stupor, high levels of vitamin D
Vitamin A (Retinol): antioxidant (neutralizes free radical), composes visual pigments promote bone resorption
(retinal; B-carotene is converted to cis-retinal which photoisomerizes to trans-retinal
when light is absorbed; cofactor for protein rhodopsin), essential for normal Vitamin E (a-tocopherol): antioxidant, prevents peroxidation of fatty acids (allows
differentiation of epithelial cells into specialized tissue (pancreatic and mucus-secreting maintain membrane fluidity), prevents oxidation of LDL, protects against arteriosclerosis
cells; binds intracellular receptors that regulate transcription at the retinoic acid o Deficiency: rare; due to fat malabsorption. Symptoms:↑ fragility of erythrocytes
response elements), immune system stimulation (vitamin A recommended for treating (hemolytic anemia), muscle weakness, neurodysfunction (poor joint sensation
measles), retinitis pigmentosa (vitamin A supplementation used as a treatment in and ataxia)
retinitis pigmentosa) o Excess: synergistic ↓ in vitamin K dependent clotting factors with warfarin
o Source: found in liver, green/yellow veggies, B-carotene (retinal dimer that must (potential hemorrhage in warfarin patients)
be cleaved and converted to trans-retinol for intestinal absorption), isotretinoin
(form of retinoid acid used to treat acne) Vitamin K: catalyzes γ-carboxylation of glutamic acid residues on blood clotting proteins
o Deficiency: due to deficient intake, malabsorption, fat-free diet. Symptoms: night (cofactor for γ-glutamyl carboxylase; allows Ca2+ binding site; co-translational
blindness, xerophthalmia, squamous metaplasia of corneal epithelium (spots are modification in RER; necessary for the synthesis of clotting factors II (prothrombin), VII,
known as Bitot spots), follicular hyperkeratosis (dry skin due to loss of IX, X, and protein C and S)
sebaceous gand), frequent infections o Must be activated by epoxide reductase enzymes (inhibited by warfarin, which is a
o Excess: due to over supplementation (>15 x RDA), consumption of wild game liver, vitamin K antagonist; an anticoagulant in vivo (not in vitro); 2-3 days required to
isotretinoin treatment. Symptoms: arthralgia (periosteal proliferation), alopecia, achieve anticoagulation thus heparin given for immediate results)
papilledema/seizure (intracranial swelling), skin changes (yellow pigment with o Source: normal gut flora, greens, breast milk doesn’t contain vitamin K
excess B-carotene but sclera remains white so not jaundice), teratogenic (cleft o Deficiency: due to fat malabsorption, neonates have sterile intestines and are
palate and cardiac abnormalities; pregnancy test must be done prior to unable to synthesize vitamin K (aggravated by mothers who took
prescribing isotretinoin). anticonvulsants during pregnancy; neonates are given vitamin K injection at
birth to prevent hemorrhage), can also occur after prolonged use of broad-
Vitamin D: raises Ca levels (↑: duodenal absorption of Ca and phosphate, reabsorption of spectrum antibiotics (destruction of normal gut flora) and↓ in hepatic function
Ca from distal renal tubules, bone resorption via activation of osteoclasts: can ↓ vitamin K activation. Symptoms: ↑ PT and normal aPTT, but normal
bisphosphonates inhibit activation of osteoclasts and ↓ bone resorption e.g., ibandronate, bleeding time (can also see increased aPTT, but as factor VII has the shortest
risedronate, and alendronate). Other functions: half-life, PT increases before aPTT), easy bruising and bleeding, hemorrhagic
o Bone Remodeling: osteoblasts have vitamin D receptors, binding stimulates disease of the newborn
release of alkaline phosphatase (alk-phos), alk-phos dephosphorylates o Excess: rare; symptoms: hemolytic anemia, liver damage
pyrophosphate which normally inhibits bone mineralization.
o Matures macrophage stem cells into osteoclasts Water Soluble Vitamins: excess is eliminated in urine (except B12 and folate that are
o Source: pre-formed ingestion in diet (D2: ergocalciferol from plants, used as stored in the liver). B-complex deficiencies result in dermatitis, glossitis, diarrhea.
supplements; D3: cholecalciferol from fish and milk) formed in sun-exposed skin o B1 (thiamine: TPP)
(UV catalyzes conversion of 7-dehydrocholesterol → cholecalciferol (D3) in skin o B2 (riboflavin: FAD and FMN)
→ 25-hydroxycholecalciferol (25-OH D3) in liver catalyzed by 25-hydroxylase) → o B3 (niacin: NAD+)
1,25-dihydroxycholecalciferol (1,25-(OH)2 D3) in kidney catalyzed by 1α- o B5 (pantothenic acid: CoA)
hydroxylase o B6 (pyridoxine: PLP)
 Decrease in hepatic function may result in vitamin D deficiency (must o B12 (cobalamin)
supplement with 25-OH D3 ; occurs in the P450 system. o C (ascorbic acid)
 1α-hydroxylase upregulated by PTH in response to hypocalcemia o Biotin
 Decrease in renal function may result in vitamin D deficiency (must supplement o Folate
with 1,25-(OH)2 D3 equivalent). Pseudo-vitamin D deficiency rickets:
hereditary deficiency in 1α-hydroxylase. Vitamin B1 (thiamine): a cofactor for several enzymes such as thiamine pyrophosphate
 25-OH D3 = storage form; 1,25-(OH)2 D3 (calcitriol) = active form (TPP), pyruvate dehydrogenase (glycolysis), α-ketoglutarate dehydrogenase (TCA cycle),
o Deficiency: due to no sun exposure, decrease in hepatic/renal function, fat transketolase (HMP shunt), branched-chain AA dehydrogenase (metabolism of Val, Leu,
Ile) synthesis of niacin from tryptophan.
o Deficiency: due to alcoholism (MC cause in US; EtOH interferes with thiamine o Deficiency: due to INH, OCP, goat milk, chronic alcoholism. Symptoms: convulsions,
absorption in small intestine), malnutrition (non-enriched rice). Symptoms: hyperirritability, peripheral neuropathy, sideroblastic anemias, cheilosis or
impaired glucose breakdown due to decreased pyruvate dehydrogenase activity stomatitis
(leads to ATP deplation, highly aerobic tissue are affected first), Wernicke’s
syndrome (ataxia, confusion, nystagmus, ophthalmoplegia), Korsakoff’s Vitamin B12 (cobalamin): cofactor for:
syndrome (confabulation, psychosis, mammillary body hemorrhage), beriberi:  Homocysteine methyltransferase: transfers CH3 groups as methylcobalamin; cofactor
o Dry beriberi: peripheral neuropathy due to for homocysteine + N-methyl THF → methionine + THF
demyelination; symmetrical muscle wasting;  Methylmalonyl-CoA mutase: metabolism of propionate (odd-chain fatty acid
no fluid retention degradation) at the conversion of methylmalonyl CoA → succinyl CoA; folate not
o Wet beriberi: high-output cardiac failure directly involved in this pathway; megaloblastic anemia with an elevated
(dilated cardiomyopathy), edema methylmalonyl CoA indicates B12 deficiency as opposed to folate
o Diagnosis: by measuring increased transketolase activity after thiamine  Other: Metabolism of Val, Met, ILe, Thr
administration (thiamine is a cofactor necessary for the function of  Source: animal products; several years’ reserve stored primarily in liver
transketolase; diagnosis of thiamine deficiency is made by history)  Deficiency: causes pernicious anemia (intrinsic factor required for absorption in the
o Treatment: first treat thiamine deficiency, then administer glucose IVF (thiamine terminal ileum; B12 not absorbed when intrinsic factor not produced from the
is a cofactor for enzymatic steps in glycolysis, administering glucose before parietal cells of the stomach), gastric bypass surgery (less intrinsic factor produced),
thiamine could further decrease thiamine levels for enzymes like transketolase resection of terminal ileum (Crohn’s disease), malabsorption (sprue, enteritis),
which could exacerbate Wernicke-Korsakoff syndrome) bacterial overgrowth of terminal ileum, diphyllobothrium latum (parasite; competes
for B12 absoroption), vegan diet. Symptoms: microcytic megaloblastic anemia (also
Vitamin B2 (riboflavin): cofactor (for oxidation and reduction (e.g., FADH2), succinate found in folate deficiency), hyperhsegmented PMNs, neurologic symptoms due to
dehydrogenase), precursor to FAD and FMN, involved with many dehydrogenase abnormal myelin (paresthesia and subacute combined degeneration, dorsal
enzymes columns of spinal cord degenerate causing loss of proprioception and vibration
o Deficiency: due to severe malnourishment; symptoms: cheilosis (inflammation sensation; not found in folate deficiencies; could be reversible with administration
of the lips and scaling and fissures at the corners of the mouth), corneal of B12; severe symptoms and longer term B12 deficiency = more residual neurologic
vascularization, dry skin, magenta-colored tongue damage and less function regained)
 Schilling test to detect etiology of the deficiency: differential process of radiolabeled
Vitamin B3 (niacin): constituent of NAD+and NADP+ (used in redox reactions), derived B12
from tryptophan, involved with many dehydrogenase enzymes, synthesis requires o Oral B12 + IM B12
vitamin B2 and B6 o B12 + intrinsic factor
o Deficiency: diets low in tryptophan or niacin (corn staple diets), Hartnup disease o B12 + antibiotics
(↓ tryptophan absorption in kidneys and small intestine), malignant carcinoid o B12 + pancreatic enzymes
syndrome (↑ tryptophan metabolism in production of serotonin), INH therapy (↓
vitamin B6 leading to ↓ niacin synthesis). Symptoms: glossitis, severe deficiency Folic acid: converted to tetrahydrofolate (THF), a coenzyme for 1-carbon
leads to pellagra (3D’s: diarrhea, dermatitis, dementia) transfer/methylation reactions; important for the synthesis of nitrogenous bases in DNA
o Excess: when nicotinic acid given at high doses as hyperlipidemic treatment and RNA (thymidylate synthase), recall: uridine + methyl group = thymidine
(raised HDL). Symptoms: facial flushing (mediated by prostaglandins, treated o Deficiency: MC vitamin deficiency in the US; absorbed in the jejunum via the
with aspirin), intrahepatic cholestasis, hyperglycemia, hyperuricemia action of intestinal conjugase. Caused by dietary insufficiency (elderly, goat
milk), alcoholism/pregnancy (liver store lasts 3 months), phenytoin,
Vitamin B5 (pantothenate): component of coenzyme A required for many enzymatic sulfonamides, methotrexate, EtOH
processes: o Findings: macrocytic, megaloblastic anemia, hypersegmented neutrophils,
 fatty acid synthase (fatty acid metabolism) homocysteinemia (↑ risk of DVT and atherosclerosis), no neurologic symptoms +
 acyl transferases normal methylmalonic acid level (as opposed to vitamin B12 deficiency),
 pyruvate dehydrogenase (PDH) deficiency in pregnancy causes fetal neural tube defects
 α-ketoglutarate dehydrogenase (TCA cycle)
o Deficiency: rare; symptoms: dermatitis, enteritis, alopecia, and adrenal Biotin: cofactor for carboxylation enzymes (adds a 1-carbon group):
insufficiency  Pyruvate carboxylase: pyruvate (3C) → oxaloacetate (4C); gluconeogenesis
 Acetyl-CoA carboxylase: acetyl-CoA (2C) → malonyl-CoA (4C); fatty acid
Vitamin B6 (pyridoxine): converted to pyridoxal phosphate, a cofactor used in: synthesis
transamination (e.g., ALT and AST in protein catabolism), decarboxylation reactions,  Propionyl-CoA carboxylase: propionyl-CoA (3C) → methylmalonyl-CoA (4C);
glycogen phosphorylase, cystathionine synthesis, heme synthesis. Required for the
odd-carbon fatty acids, Val, Met, Ile, Thr catabolism o Reactions happen frequently, but the body can defend itself with anti-oxidants:
o Deficiency: rare; due to antibiotic use, excessive ingestion of raw eggs (contains oxidative stress occurs when pro-oxidants > anti-oxidants.
avidin which binds biotin). Symptoms: dermatitis, alopecia, enteritis, lactic o Examples of oxidative stress damage: stroke, Parkinson’s, Alzheimer’s
acidosis
Iron: source: diet, recycled erythrocytes. Forms: ferrous iron (Fe2+; dangerous, causes
Vitamin C (ascorbic acid): antioxidant (regenerates vitamin E; ↓ oxidation of LDL), keeps oxidative stress, found in Hb), ferric iron (Fe3+, less dangerous, methemoglobinemia
iron in Fe2+ reduced state (↑ intestinal absorption), collagen synthesis (essential for (metHb) occurs when Fe3+ is found in hemoglobin).
hydroxylation of proline and lysine by prolyl and lysyl hydroxylases; addition of o Absorption: Fe-containing compounds are solubilized in low stomach pH. Fe3+ is
hydroxyl group allows for hydrogen bonding between fibers, without cross-linking triple reduced to Fe2+ (requires vitamin C) in intestine so it can cross gut lumen.
helix shape cannot form), synthesis of norepinephrine (necessary for dopamine β- Ferroportin brings Fe3+ into bloodstream from enterocytes, mediates amount of
hydroxylase which converts dopamine to NE), hepatic synthesis of bile acids, keeps THF Fe released into the blood, hepcidin inhibits ferroportin (antibacterial because it
in reduced form, protects against nitrosylation of amides (occurs in the stomach with lowers the availability of iron in the plasma).
presence of food preservatives; nitrosamines/amides are carcinogenic) o Storage: must be immediately used or stored to prevent: bacterial utilization (Fe
o Source: fruits and vegetables; british sailors used to carry limes to prevent required for growth), formation of iron oxides, free radicals (Fe + O2). Site:
scurvy hepatocytes (main), enterocytes, macrophages. Stored as ferritin (Fe3+).
o Deficiency: diet lacking citrus fruits and greens, infants on formula that is boiled Hemosiderin binds excess Fe3+ to prevent from entering the blood.
too long (infantile scurvy; 2-10 months; excessive heat destroys vitamin C), o Transport: carried as Fe3+ by transferrin in the blood (transferrin chelates the Fe3+
smoking. Symptoms: scurvy (swollen gums, bruising, perifollicular hemorrhage, and transports it in the blood to tissues; maintains solubility and keeps
poor wound healing, glossitis,↑ bleeding time, anemia due to combined iron and unreactive, transferrin receptors on cells endocytose transferrin:Fe complex,
folate deficiency. Fe3+ released into cell triggered by low pH, transferrin returns to cell surface to
o Excess: causes renal calculi made from calcium oxalate (vitamin excreted as be used again), ferroxidase (aka ceruloplasmin) oxidizes Fe2+ to Fe3+ for
oxalate), diarrhea, nausea, vomiting, excess iron absorption in those predisposed transport and storage (ferritin can also oxidize Fe for storage).
(hemochromatosis, repeat blood transfusions) o Excretion: no cellular mechanism for iron excretion; lost from blood loss and
removal of skin cells and other epithelial cells
S-Adenosylmethionine (SAM): Not a vitamin but an important cofactor. o Toxicity: beyond the sequestration capacity of ferritin; causes oxidative stress
o Synthesis: ATP + methionine = SAM; regeneration of methionine (and thus SAM) is o Disorders in iron handling: hereditary hemochromacytosis
dependent on vitamin B12 and folate
o Function: SAM transfers methyl units (similar to THF); required for the conversion Copper: role: human metabolism. Like other metals, free copper is potentially toxic by
of NE to epinephrine donating electrons (creates hydroxyl radicals and other reactive oxygen species), is a
cofactor for many metalloproteins (eg. lysyl oxidase for collagen synthesis and
Kwashiorkor: due to protein-deficient diet. Presentation: skin lesions, pitting edema (↓ tyrosinase for melanin synthesis)
albumin leads to ↓ oncotic pressure in vasculature and loss of fluid into extravascular  Transport: albumin and ceruloplasmin carry copper in the blood (similar role to
space), ascites, liver malfunction (↓ apolipoprotein synthesis; fatty liver),defects in cell- transferrin in iron transport), metallothionein is a carrier of copper, zinc, and many
mediated immunity (↓ in complement protein synthesis), muscle protein relatively other metals (role in preventing oxidative stress in the cell; thiol groups from many
unchanged. cysteine residues mediate binding)
 Excretion: excess copper removed in the bile unlike iron with no mechanism of
Marasmus: due to protein- and caloric-deficient diet. Presentation: tissue and muscle excretion
wasting (broomstick extremities; breakdown of muscle protein for energy), loss of  Deficiency: causes: excess zinc (metallothionein carries both copper and zinc; copper
subcutaneous fat, variable edema. is displaced when zinc concentrations rise). Symptoms: a function of what enzymes
require copper (ferroxidase: catalyzes oxidation of iron from Fe2+ to Fe3+,result is
Trace metals: micronutrients required in the diet for necessary cellular functions: iron, microcytic anemia; lysyl oxidase: crosslinks collagen fibers, result is poor wound
copper, zinc, chromium, fluoride, iodide, selenium, etc. healing, aortic dissection)
o Function: part of metalloenzyme (enzyme has no activity without the metal. Metal  Disorders in copper handling: Wilson’s disease, Menke’s disease: X-linked gene
is fixed, metal:protein is constant. Eg: carbonic anhydrase) and is a part of metal- mutation in ATP7A (ATP-dependent copper efflux protein); aka Ehlers-Danlos
containing enzyme (enzyme may have activity without the metal; metal is syndrome type IX; inability of enterocytes to release absorbed copper; copper at
reversibly bound, metal:protein ratio is variable. Eg. glycogen phosphorylase toxic levels in small intestine and kidneys; copper in circulation and in brain at low
kinase) levels. Symptoms: presents like a copper deficiency: seizures, failure to thrive,
o Oxidative stress: organometallic side reactions that damage tissue. Many metals neurodegeneration, steel-colored and brittle hair (due to role of copper in
undergo the Fenton reaction in vivo (oxidation of metal and donation of an metalloprotein lysyl oxidase which crosslinks collagen for added strength; at low
electron to oxygen; most common metals that undergo reaction are Fe2+ and Cu+; serum concentrations of copper, this enzyme cannot function).
creation of hydroxyl radicals). Heme iron can generate superoxide radicals (O2*)
asparagine (amino-group), O-oligosaccharides on serine and threonine
Zinc: hundreds of enzymes require zinc (carbonic anhydrase, ACE (angiotensin I (hydroxyl-group), mannose-6-phosphate designates transport to lysosome
converting enzyme), RNA and DNA polymerase etc) (deficient in I-cell disease), sulfation of tyrosines) and cleavage of propeptides
 Transport: metallothionein carries zinc (competes with copper) (peptidase cleaves proinsulin to insulin + C-peptide, is deficient
 Deficiency: due to: poor diet, alcoholism (liver is unable to handle zinc properly). in hyperproinsulinemia: presents similar to NIDDM)
Symptoms: impaired collagenase (delayed wound healing), impaired zinc finger o Packaging/distribution of proteins and lipids received from ER: (adds amino acid
transcription factor motifs (hypogonadism; ↓ adult hair: axillary, facial, and pubic), ↓ residues that golgi later modifies) via coat protein complex (COP) II: this protein
in senses (dysgeusia: lack of taste and anosmia: lack of smell), diarrhea, hair loss initiates budding process (RER→ cis-Golgi = anterograde, anterograde
(alopecia) movement mediated by COP II; Golgi → ER = retrograde, retrograde movement
mediated by COP I) leaves Golgi to several destinations: plasma membrane,
Chromium: deficiency: due to total parenteral nutrition (TPN). Symptoms: a function of lysosome, secretory vesicles, mediated by clathrin
what proteins/enzymes require chromium (hypothesized to play a role as part of glucose o Proteoglycan assembly (sulfation of sugars).
tolerance factor): ↓ glucose tolerance
Lysosome: function: digest endosomal material (via digestive enzymes: glycosylases,
Fluoride: source: fluoridated water; plays a role in thyroid hormone synthesis. lipases, proteases; all are acid hydrolases which function at low pH); are found in all cells
Deficiency: goiter, ↓ thyroid hormone output (higher concentrations in phagocytic cells). Forms: primary lysosomes (newly formed
from the trans-Golgi, waiting to receive endocytosed material) and secondary lysosomes
Selenium: plays a role in glutathione peroxidase that protects against oxidative stress (aka phagolysosome; formed when primary lysosomes fuse with endocytic vesicles).
thus damage to tissues with high metabolic activity. Deficiency: due to TPN; muscle pain, Lysosomal storage disease (by deficient enzymes):
cardiomyopathy  Sphingolipidoses:
o Sphingomyelinase; deficient in Niemann-Pick disease, AR; common in Ashkenazi
Cellular Biochemistry: Jews; sphingomyelin accumulates (histiocytes look “foamy”). Presentation:
hepatosplenomegaly, anemia, cherry red spot on macula, death <3yrs, FTT,
RER: composed of a lipid membrane with membrane bound ribosomes (lipid membrane neurodegeneration.
is continuous with nuclear membrane) o α-galactosidase A: deficient in Fabry disease, XR; ceramide trihexose accumulates.
o Function: synthesis of secretory proteins (e.g. peptide hormones) and addition of Presentation: peripheral neuropathy (especially in hands and feet),
N-linked oligosaccharides to peptides angiokeratomas (small purple blemishes on skin), cardiovascular disease, renal
o Found in high concentration in: neurons (termed Nissl bodies; stain basophilic; disease
synthesize/secrete peptide neurotransmitters in prepackaged vesicles), o β-galactocerebrosidase: deficient in Krabbe disease, AR;
pancreatic acinar cells (synthesize/secrete digestive enzymes) galactocerebroside accumulates. Presentation: hyperactive reflexes, optic
atrophy, developmental delay
SER: structure: lipid membrane without membrane bound ribosomes. o β-glucocerebrosidase: deficient in Gaucher disease, AR, common in Ashkenazi Jews;
 Function: steroid synthesis, detox of chemicals (makes compounds water soluble by glucocerebroside accumulates in cells of phagocytic cells (histiocytes (dendritic
two mechanisms hydroxylation via cytochrome P450 hydroxylase complex and cells) look like wrinkled tissue paper called Gaucher's cells). Presentation: three
conjugation which involves binding of polar moiety (e.g. glucuronate via glucuronyl types:
transferase) to toxin), lipid metabolism (release of fatty acids from triglycerides,  Type I: MC; hepatosplenomegaly, aseptic necrosis of heads of long bones, mild
assembles lipoproteins for release), carbohydrate metabolism (gluconeogenesis: anemia, possible to live a normal lifespan
allows free glucose to be released into circulation during fasting by removing  Type II: "infantile Gaucher”; CNS involved, death <1yr
phosphate from glucose-6-phosphate mediated by glucose-6-phosphatase; deficient  Type III: "juvenile Gaucher”; severity < type II
in von Gierke's disease: glycogen storage disease type I, presents with o Hexosaminidase A: deficient in Tay-Sachs disease, AR, common in Ashkenazi Jews;
accumulation of glycogen in kidney and liver (hepatomegaly) and hypoglycemia). GM2 ganglioside accumulates, lysosomes with onion skin. Presentation: CNS
 Found in high concentration in: hepatocytes, kidney, adrenal cortex, corpus luteum, degeneration, blindness, cherry red spot on macula, startle reflex, death <4yrs,
muscle (modified SER: sarcoplasmic reticulum which stores and releases Ca to similar to Niemann-Pick but without hepatosplenomegaly.
mediate muscle contraction). o Arylsulfatase A: deficient in metachromatic leukodystrophy, AR; cerebroside
sulfate accumulates. Presentation: demyelination in CNS and PNS resulting in
Golgi Apparatus: structure: stacks of cisternae, associated with membrane bound ataxia and dementia
vesicles, has two faces: cis (receiving face from the ER, input to the Golgi, concave shape)  Mucopolysaccharides:
and trans (exports away from Golgi, location of trans-Golgi network, faces plasma o α-L-iduronidase: deficient in Hurler syndrome, AR; heparan sulfate and dermatan
membrane, convex shape). Function: sulfate accumulates in heart and liver. Presentation: gargoyle-like facies, corneal
o Post-translational peptide modification: (addition of N-oligosaccharides on clouding, progressive mental retardation
o Iduronate sulfatase: deficient in Hunter syndrome, XR; heparan sulfate and
dermatan sulfate accumulates. Presentation: severity < than Hurler’s,
aggressive behavior (remember: hunter's are aggressive), corneal clouding Plasma membrane: structure: bilayer of phospholipids, asymmetrical in respect to
absent intracellular and extracellular faces, ”fluid mosaic”. Composition:
 Cholesterol: 50%; adds thermal stability to membrane (↑ melting temperature), ↓
Other lysosomal disorders: membrane flexibility; amount in plasma membrane tightly regulated
 I (inclusion)-cell disease: cause: proteins marked for localization to lysosomes are  Phospholipids: 50%; sphingolipids: fatty acid chain attached to a sphingosine.
post-translationally modified in the Golgi. Mannose residues are phosphorylated by Disorders of sphingolipid metabolism: Fabry’s and Gaucher’s disease.
N-acetylglucosamine-phosphotransferase enzyme, defect in this enzyme causes I-  Glycolipids
cell disease. Without mannose-6-phosphate designation, the enzymes are secreted  Proteins:
instead of being targeted to the lysosome. Cells cannot degrade endocytosed  Pumps: move substances against their concentration gradient, requires energy. E.g.
material and inclusion bodies build up intracellularly. Na+-K+ ATPase:
o Presentation: high plasma levels of lysosomal enzymes, skeletal abnormalities, o Transmembrane pump, ATP binding-site accessible from cytoplasm, process: 3 Na+
restricted joint movement, psychomotor retardation, early death, coarse facial bind pump intracellularly, ATP binds and phosphorylates the pump, pump
features. No treatment exists. changes conformation which releases 3 Na+ extracellularly, 2 K+ bind pump
 Chédiak–Higashi syndrome: AR; cause: primary lysosomes of leukocytes cannot fuse extracellularly, phosphate ion removed. Pump changes conformation which
with phagosomes; due to inability of microtubles to polymerize; also affects immune releases 2 K+ intracellularly, 3 Na+ bind pump intracellularly (repeat). Net: each
cell chemotaxis. Presentation: ↑ infections (especially S. aureus), partial albinism, ATP results in 3 Na+ out and 2 K+ in.
peripheral neuropathy. o Pharmacological importance: cardiac glycosides (digoxin and digitoxin; mechanism
of action: inhibits pump, depolarization of cell membrane, ↑ intracellular [Na+], ↓
Peroxisome: structure: membrane-bound vesicle, contain several important enzymes Na+ gradient required for Na+/Ca2+ exchange, ↑ [Ca2+] intracellularly, ↑ cardiac
(catalase, hydrogen-transferring enzymes etc). contractility) and ouabain (inhibits by binding to K+ site, similar response to
 Deficient peroxisome assembly: infantile refsum disease, AR; brain disorders, skeletal cardiac glycosides)
and craniofacial dysmorphism, liver dysfunction, common presentation to all  Channels: move substances down their concentration gradient. 3 types:
peroxisomal disorders  Ungated: always open (eg. K channel)
 Function:  Voltage gated: open in response to changes in membrane voltage (found in excitable
 Degradation of very long chain fatty acids (VLCFAs): > 24 carbons; via β-oxidation; tissue)
reduces by 14 carbons and then sends to mitochondria  Ligand-gated: open in response to a ligand (e.g. post-synaptic membrane receptors)
o Deficient in Zellweger syndrome: AR; build-up of VLCFAs in peroxisomes, impaired  Function: selective permeability (controls an intracellular environment distinct from
myelin synthesis (severe neurological symptoms), hepatomegaly, death < 1 year extracellular environment), signalling, localization of enzymes to promote or inhibit
(can increase lifespan with diet low in VLCFAs). Other examples of peroxisomal interaction
disorders: neonatal adrenoleukodystrophy (NALD) Membrane physiology:
 Bile acid synthesis: derived from cholesterol  Electrochemical potential determined by: conductance (G; ability of ions to
 Phospholipid modification: alter phosphatidylserine and phosphatidylethanolamine move across a membrane; controlled by opening/closing channels, ↑ channels =
 Degradation of hydrogen peroxide: via catalase degrades hydrogen peroxide produced ↑ conductance) and net force (combination of concentration force:
in β-oxidation of fatty acids concentration difference of a substance across a membrane and electrostatic
force: attraction of unlike charges/repulsion of like charges)
Mitochondria: found in high concentrations in metabolically active cells. Structure:  Equilibrium potential: defined as the electrical potential across a membrane
double membrane structure: outer membrane (high permeability due to high that would prevent the diffusion of a substance via its concentration force for a
concentration of porins) and inner membrane (impermeable to ions and molecules; given concentration difference across a membrane
folded into cristae to ↑ surface area; contains enzymes for electron transport chain). o Measured in millivolts (mV)
Inner matrix contains circular DNA (mtDNA). o For a single substance, calculated by: Nernst equation: Ex+ = 60/Z log(
 Maternal inheritance as mitochondria in the zygote come from the egg. [X+]extracell / [X+]intracell ); Z = absolute value of ionic charge; K+, Cl-, Na+ =
 Non-Mendelian inheritance examples of mtDNA disorders: Leber’s optic neuropathy 1, Ca2+ = 2. Answers the question: what is the voltage that exists across a
(degeneration of retinal ganglion cells, progressive loss of central vision, eventual membrane when a certain ion is at its equilibrium. Another way of
blindness), Pearson marrow-pancreas syndrome (some protein synthesis occurs thinking of this: what is the voltage required so that there will be no
here; most proteins are synthesized in cytoplasm and translocated to mitochondria) net flow of a certain ion? For example: -80 mV is the Nernst potential
 Function: oxidizes substrates (NADH/FADH2 → NAH+/FAD), generates ATP (converts for potassium. This means that if the inside of the cell was -80 mV, K+
NADH/FADH2 into ATP via ETC), stores Mg2+ and Ca2+ would not leave or enter the cell. The -80 mV of the cell pulling K+ in is
 Mitochondrial inheritance pedigree: Affected males (transmit to none of their equal to the concentration gradient that wants to pull K+ out (remember
children), affected females (transmit to all of their children) that K+ is low outside the cell and wants to travel down its ion gradient).
If the voltage became -81 mV then K+ would want to travel into the cell as to dynein arm defect. Presentation: male and female infertility: sperm
this negative charge would overpower the drive for K+ to travel down its are immotile (no functional flagellar tail), fallopian tubes cannot
concentration gradient out of the cell. If the voltage was -79 mV then K+ sweep egg and sperm towards each other; bronchiectasis, recurrent
would leave the cell as the concentration gradient pulling K+ out is sinusitis (mucus with bacteria and particles cannot be removed;
greater than the negative voltage attracting/holding K+ in associated with situs inversus)
o Does NOT determine rate of ionic diffusion → only whether diffusion is • Mitotic spindles
favorable • Molecular motor proteins, mediates intracellular transport. 2 types: kinesin (cell
 Resting membrane potential (Emem): equilibrium potential of most cells = -90 center → periphery; anterograde to microtubule e.g. transports
mV; calculated by: sum of individual membrane potentials for neurotransmitter vesicles down axon towards synapse) and dynein (periphery
all permeable ions proportional to their conductances, for ions X+, Y+, and Z: → cell center; retrograde to microtubule e.g. lipid transport from synapse back
Emem = Gx(Ex+) + Gy(Ey+) + Gz(Ez-). Note: the closer the resting membrane to Golgi apparatus)
potential is to the equillibrium potential of an individual ion, the greater the • Pharmacologic importance:
membrane conductance is for that ion. When Emem = Ex+ , there is no net • Mebendazole: antihelminthic; mechanism of action: ↓ microtubule synthesis in worms
movement of ions and net force = 0. Example: • Griseofulvin: antifungal; moa: deposits in new keratin and disrupts microtubule
 Emem = -77 mV polymerization; uses: active against dermatophytes only
 EK+ = -95 mV • Vincristine/vinblastine: anti-cancer; moa: ↓ microtubule polymerization, inhibits
mitosis
Type Filament name Locations Stain • Paclitaxel (taxol): anti-breast cancer; moa: ↑ stability of microtubule and does not
I-II Keratin Epithelial cells Cytokeratin allow disassembly; inhibits mitosis
III Desmin Muscle (smooth, cardiac, and skeletal) Desmin • Colchicine: anti-gout; moa: binds free tubulin, ↓ microtubule polymerization, inhibits
Peripherin Peripheral nerve axons - leukocyte/granulocyte migration
Vimentin Fibroblasts and endothelium Vimentin
Glial fibrillary acidic Astrocytes and Schawnn cells GFAP Intermediate filaments:
protein (GFAP) • Structure: stable polymers; unlike microtubules and actin → does not undergo
IV Neurofilaments Neurons Neurofilament dynamic treadmilling; must be degraded via the ubiquitin pathway (sends to
(NF) proteasomes). Intermediate in size between microfilaments (7 nm) and
V Lamins Nuclear envelope of all cells - microtubules (25 nm)
 Is diffusion of K+ across this membrane favorable? Yes, given open • Function: give strength to the cytoskeleton: 5 types:
channels (G) to K+ it will diffuse until Emem = -95 mV  Clinical importance: alcoholic liver disease: ubiquitinated cytokeratin filaments
o Inside cells (as compared to extracellular environment) accumulate in hepatocytes (eosinophilic inclusions called Mallory bodies) and
 ↑ K+: EK+ = -95 mV; G is high for K+→ changes in [K+]extracellular will have parkinson's disease: alpha-synuclein and ubiquitinated neurofilaments accumulate
a large impact on Emem. ↑ G will hyperpolarize cell (efflux from cell) in neurons of substantia nigra eosinophilic inclusions called Lewy bodies
 ↓ Na+: ENa+ = +45 mV; G is low for Na+ → changes in [Na+]extracellular will
NOT have a large impact on Emem. ↑ G will depolarize cell (influx into Microfilament: examples: actin:
cell)  Structure: helical polymers of G-actin; has polarity (+ end is site of formation, - end
 ↓ Cl-: ECl- = -90 mV; since in most cells Emem = -90 mV → Cl- is is site of degradation); "treadmilling": formation requires ATP; dynamic structure
at equilibrium and will not diffuse that is constantly forming and breaking down , can sample environment, G-actin has
intrinsic ATPase capabilities.
Microtubule: cellular structural protein with a hollow tubular structure  Function: can interact with myosin to mediate: contractile force, motile force,
• Structure: composed of polymerized dimers of α- and β-tubulin, each dimer has 2 GTP cytokinesis and can interact with cell membrane to: form core of microvilli (function
molecules bound; constant assembly (slow) and disassembly (fast) to increase absorptive surface area), anchor tight junction and zonula adherens
• Clinical importance: Chédiak–Higashi syndrome (CHS): AR; disease caused by a
microtubule polymerization defect resulting in decreased chemotaxis, Cell surface protein: purpose: allow cells to attach to: each other (via different types of
degranulation, phagocytosis. Presentation: recurrent pyogenic infections cell-cell junctions), basement membrane (complex interaction between cells and
(particularly S. aureus), partial albinism, peripheral neuropathy supporting matrix, two divisions: basal lamina (provides attachment for most types of
• Function: component of many important cellular structures: epithelium; composition: type IV collagen, laminin, heparan sulfate) and reticular lamina
• Cilia: 9+2 arrangement of microtubules; axonemal dynein: ATPase that attaches the (supports lymphoid and adipose tissues; composition: type III collagen [reticular
peripheral 9 doublets causes bending of cilium by binding differentially to fibers]). Types of molecules:
doublets; also forms the core of flagella
◦ Clinical importance: Kartagener syndrome: immotile cilia disease due
 Cadherin: structure: calcium dependent, binds cadherin dimer on another cell  Gap junctions: structure: pores formed by 6 connexon proteins
extracellularly and actin intracellularly (via catenin). Function: ↑ cell-cell o Function: allow direct passage of small molecules from one cell to another (e.g.
binding (cadherin receptor ↓ regulated in cancer metastasis) Ca2+, cAMP); role in electrical and metabolic signaling; no role in strength
 Selectin: calcium dependent. Function: binds carbohydrates in cell-cell
interactions. Subtypes: Extracellular matrix:
o L-selectin (on leukocytes): have Sialyl-Lewis glycoproteins as ligands  Component: collagen, elastin, and:
o P-selectin (on platelets and endothelial cells)  Glycoprotein: structure: protein + carbohydrates (protein content > carbohydrate
o E-selectin (on endothelial cells): binds leukocytes strongly, ↑ content; opposite true for proteoglycans), function: attach cells to various
expression on surface during acute inflammatory components of the ECM. Examples: entactin, tenascin, laminin
response (stimulated by TNF-α)  Fibronectin: structure: insoluble monomers, functions: attach cells to various
 Integrin: calcium-independent, transmembrane; binds fibronectin and laminin components of the ECM; can circulate in plasma to assist with clotting; biofilm
extracellularly and actin intracellularly. Function: binds leukocytes formation via cell adhesion to basement menbranes
weakly (neutrophilic attachment in rolling via an integrin LFA-1, defective in  Proteoglycans: structure: protein + glycosaminoglycan (GAGs). GAGs have large
leukocyte adhesion deficiency (LAD) type I: presents with recurrent bacterial amount of negative charges due to sulfation, examples of GAGs: chondroitin sulfate,
infections and delayed loss of umbilicus postpartum) and bind to laminin in dermatan sulfate, keratan sulfate, heparan sulfate, hyaluronic acid. Function: high
ECM (integrin receptor ↑ regulated in cancer metastasis) degree of hydration; sponge-like nature resists compression; found in high
concentrations in cartilage
Types of cell junctions:
 Tight junctions: aka zonula occludens. Structure: zona occuldens (ZOs) 1, 2, 3; Collagen: Synthesis and structure:
claudin proteins; membrane spanning proteins; bind cellA actin intracellularly on  Inside fibroblasts:
one end, bind cellB actin intracellularly on opposite end o pre-pro-collagen α chain formation: RER-bound ribosomes synthesize; contains
o Location: apical end of epithelial cells hydrophobic translocation sequence; chain formed mainly of repeating tripeptide
o Function: prevent diffusion, create cell membrane polarity o Gly-X-Y: X and Y are proline, lysine
 Adherens junctions: aka zonula adherens. Structure: cadherin proteins (Ca2+- o pro-collagen α chain formation: hydrophobic sequence cleaved
dependent adherin = cadherin) o hydroxylated pro-collagen α chain formation: X and Y position prolines and
o Location: belt around cell, below tight junction lysines are hydroxylated to form hydroxylysine and hydroxyproline. these amino
 Desmosomes: aka macula adherens. Structure: cadherin proteins; link between two acids are unique to collagen; hydroxylated as peptide chain passes into ER;
cells; attach to intermediate filaments keratin and desmoplakin proteins performed by prolyl and lysyl hydroxylase (lack of lysyl hydroxylase function
o Location: distinct sites results in weak collagen chains); requires ascorbic acid (Vitamin C): lack of
o Function: rivets; gives strength to junction between cells vitamin C causes scurvy: presentation: swollen gums, bruising, anemia, poor
o Clinical importance: pemphigus vulgaris: pathophysiology: auto-IgG against wound healing; can also be caused by defective lysyl hydroxylase gene: Ehlers-
desmosomal proteins in keratinocytes; type II hypersensitivity. Presentation: Danlos syndrome: nine different types, lysyl hydroxylase gene deficiency is one of
painful, flaccid vesicles form on skin and oral mucosae; located above basal many causes, presentation: hyperextensible skin, hyperflexible joints, weak vessel
layer, because it is above the basal layer the blister is weak; positive Nikolsky walls (↑ risk for aneurysm)
sign: outer epidermis separates with gentle rubbing; acantholysis (loss of o glycosylated pro-collagen α chain formation: hydroxylysines are glycosylated
connection between cells); post-inflammatory hyperpigmentation. Treatment: o pro-collagen α chain trimer formation: three α chains associate, moved from RER
immunosuppression (corticosteroids) to Golgi, secreted out of the fibroblast
 Hemidesmosomes: structure: composed of integrins; bind type IV collagen,  Outside fibroblasts:
fibronectin, laminin of basal lamina extracellularly and intermediate filaments o collagen molecule (tropocollagen) formation: propeptides cleaved from ends and
intracellularly becomes insoluble; presence of propeptide does not allow assembly
o Function: like a desmosome but instead of attaching cell-to-cell it attaches cell- intracellularly
to-basement membrane; "half of a desmosome" o collagen fibril formation: catalyzed by lysyl oxidase: covalently links α chains by
o Clinical importance: crosslinking hydroxylysines, copper required as cofactor (lack of copper results
o Bullous pemphigoid: pathophysiology: auto-IgG against basement membrane from Menke’s disease, at low serum concentrations of copper this enzyme cannot
hemidesmosomes; type II hypersensitivity, can be drug-induced function and weak collagen is formed, cause: X-linked gene mutation in ATP7A:
o Presentation: wide distribution of skin blisters, unlike pemphigus vulgaris → ATP-dependent copper efflux protein aka Ehlers-Danlos syndrome type IX:
does not involve oral mucosae; vesicles below the epidermis, stronger vesicles; inability of enterocytes to release absorbed copper, copper at toxic levels in small
negative Nikolsky sign; NO acantholysis. Treatment: immunosuppression intestine and kidneys, copper in circulation and in brain at low levels,
(corticosteroids) presentation: presents like a copper deficiency: seizures, failure to thrive,
o Cancer: metastasis involves timely ↓ regulation of hemidesmosomal proteins neurodegeneration, steel-colored and brittle hair
o collagen fiber formation: fibrils aggregate to form final bundles of triple helix without AAT deficiency usually causes centriacinar emphysema, cirrhosis
quaternary protein structure.
Marfan syndrome: Almost exclusively AD; mutation in FBN gene (encodes fibrillin-1 gene
Types of collagen: on chromosome 15). Patients are susceptible to aortic disease, MVP, lens dislocation,
 Type I: thick, rope-like bundles of collagen; strongest tensile form of collagen; scoliosis, pectus deformity, arachnodactyly
majority of collagen in the body (approx. 90%); found in locations where high
tensile strength is needed (bone, fascia, tendons, teeth (dentin), cornea, skin); type A1-antitrypsin deficiency: Autosomal codominant inheritance; α1-antitrypsin inhibits
III of early wound repair converted to type I in late wound repair; defective in elastase; can lead to panacinar emphysema and hepatic disease
osteogenesis imperfecta (OI) type I: aka brittle bone disease; AD, in most cases,
presentation: multiple fractures with minimal force (first fractures may occur Cell cycle: stages:
during delivery), blue sclerae (due to the translucency of the connective tissue over  Interphase (G1): purpose: cellular growth to prepare for DNA replication (synthesis
the choroid due to lack of collagen), deafness (50%; abnormal middle ear bones), of replication proteins, cyclin D), thymidine dimer repair. Variable duration, can exit
dental abnormalities; defective in various forms of Ehlers-Danlos syndrome G1 and enter G0, chemotherapeutic agents that target G0 (cisplatin, nitrosoureas,
antitumor antibiotics, alkylating agents)
 Type II: spongy collagen to absorb shock; found in tissues where there are
compression forces; cartilage (including hyaline), vitreous body of the eye, nucleus  S: purpose: DNA replication (cell is 2N before S, cell is 4N after S), DNA
pulposus of vertebral disc proofreading. Constant duration, chemotherapeutic agents that target this
stage (etoposide, 6-mercaptopurine, 6-thioguanine, methotrexate, cytarabine,
 Type III: web-like fibers where forces pull from many directions; aka reticulin; found hydroxyurea)
in tissues where strength is needed (but not compression or tensile): skin, blood
vessels, uterus, fetal tissue; granulation tissue (type III of early wound repair  G2: purpose: cellular growth to prepare for cell division, mismatch repair. Variable
converted to type I in late wound repair); defective in Ehlers-Danlos type IV: faulty duration, chemotherapeutic agents that target this stage (bleomycin)
collagen synthesis associated with: joint dislocation, berry aneurysms, organ
rupture  Mitosis (M): steps: prophase, metaphase, anaphase, telophase. Chemotherapeutic
agents that target this stage (vinblastine, vincristine, paclitaxel; all drugs that affect
 Type IV: basement membrane; especially kidney, ears, eyes, skin; organizes/solidifies microtubules)
cellular structure; defective in Alport's syndrome: effects the tissues where type IV
is most prominent (kidney → progressive hereditary nephritis, ears → deafness, Regulation: control transitions between phases of cell cycle.
eyes → ocular disturbances), majority of cases are X-linked dominant; one of the  G1 checkpoint: at transition from G1 to S; most important checkpoint
causes of epidermolysis bullosa: weak union of dermis and epidermis of the skin,  Cyclins/cyclin-dependent kinases: Cyclin D: produced during G1, complex with Cdk4
easily formed blisters; Goodpasture's syndrome involves an auto-antibody against (cyclin dependent kinase) at sufficient concentration; signal to enter S phase
collagen type IV in pulmonary and glomerular capillaries; presents with hemoptysis  Tumor suppressors: requires mutation in both copies before neoplasia occurs as
and glomerular disease opposed to oncogenes which require just one mutation
 Rb: prevents cell from entering S phase; binds E2F to block its function as a
Elastin: transcription factor; named for retinoblastoma
 Structure: rich in proline, lysine, glycine; unlike collagen, can exist in  P53: prevents cell from entering S phase; leads to inhibition of kinase activity of
nonglycosylated forms; fibrillin protein binds to tropoelastin to form elastic fibers: Cdk4 via p21; can induce apoptosis if cell damage is severe via the BAX gene
Marfan's disease is caused by a defect in fibrillin, presentation: long extremities (proapoptotic; inhibits BCL2 antiapoptosis gene; stimulates release of cytochrome c
including fingers, scoliosis, myopia and lens dislocations (upward like a martian from mitochondria)
leaving for outer space as opposed to a downward dislocation in homocystinuria);
mitral valve prolapse (↑ for aortic aneurism) Cell types:
 Function: is an elastic protein; found in tissues where stretch is needed (lungs, dermis  Permanent: enter G0 and cannot leave e.g. neurons, skeletal, cardiac muscle, RBCs
of the skin, large arteries, elastic ligaments, vocal cords, ligamenta flava of  Stable (quiescent): enter G0 and can leave when given appropriate stimulus e.g.
vertebrae); desmosine interchain cross linking between lysine residues gives the hepatocytes, lymphocytes labile; never go to G0; constant division with a condensed
protein its elastic stretch G1 e.g. bone marrow, skin, gut epithelium
 Degraded by elastase, α1-antitrypsin normally inhibits elastase (class of protease
inhibitors synthesized in the liver) Signal transduction:
 Excess elastase activity caused by α1-antitrypsin (AAT) deficiency (absent α1-globulin  Tyrosine kinase signaling: Tyrosine kinase structure: dimeric transmembrane protein,
peak in serum protein electrophoresis), autosomal codominant inheritance, intrinsic kinase activity, phosphorylated receptor can bind protein called insulin
presentation: panacinar emphysema (worsened by smoking, early onset), smoking receptor substrate (IRS; is a scaffold protein)
 Pathway: hormone binds to receptor, receptor is dimerized, auto cross- Glycolysis:
phosphorylation of intracellular domain of the dimerized receptor, IRS
binds phosphorylated domain, IRS is phosphorylated on SH2-domains. Several Function: generation of ATP from glucose via substrate-level phosphorylation (as
enzymes bind phosphorylated SH2-domians: opposed to oxidative phosphorylation). Used by all cells:
 Phosphatases: pathway signaled for by insulin  With O2: pyruvate enters citric acid cycle (after being created in glycolysis),
 Kinases: phosphoinositol-3 (PI-3) kinase NAD+ regenerated via oxidative phosphorylation, produces 4 ATP per glucose
 G proteins: ras: oncogene present in many cancer; constitutive activation leads to molecule. Without O2: pyruvate cannot enter citric acid cycle after glycolysis,
constant growth signal. Examples: insulin, PDGF, EGF receptors NAD+ must be regenerated via conversion of pyruvate to lactate catalyzed by
lactate dehydrogenase, produces 2 ATP per glucose molecule.
G protein signaling: G-protein structure: trimeric enzyme (α, β, γ), α-subunit has intrinsic  Only source of energy for RBCs (generation of intermediates for other
GTPase activity (acts as timer for shutting off the active form) pathways: 1,3-BPG (intermediate of glycolysis, can be converted to 2,3-DPG,
 Pathway: hormone binds to receptor:G-protein complex on cell membrane, GDP modifies the hemoglobin-O2 binding curve, binds HbA and ↓ binding affinity of
bound to inactive α-subunit is exchanged for GTP, α-subunit now active (can O2: a compensatory mechanism for ↓pO2)
be inhibitory (Gi) or stimulatory (Gs, Gq)), is released from β, γ subunits; can act as 2
different effector mechanisms: Pathway: in cytoplasm; irreversible; net reaction: glucose + 2Pi + 2 ADP + 2 NAD+ → 2
 cAMP pathway: Gs activates adenyl cyclase: ATP → cAMP, cAMP can activate protein pyruvate + 2 ATP + 2 NADH + 2H+ + 2 H2O
kinase A, ADP-ribosylated by cholera and E. coli toxins (activates Gs, ↑↑↑ [cAMP]).
Gi inhibits adenyl cyclase: ADP-ribosylated by pertussis toxin (inhibits Gi, Important enzymes:
↑↑↑ [cAMP])  Hexokinase: converts glucose into glucose-6-phosphate allowing "trapping" inside
 PIP2 pathway: Gq activates phospholipase C: PIP2 → IP3 + DAG, DAG can activate cell. Distribution: widely present in most body tissues, allows trapping of glucose at
protein kinase C, IP3 can release Ca2+ from the ER (Ca2+ then activates a number of all blood glucose levels. Kinetics: high affinity → low Km, low capacity → low Vmax.
enzymes including protein kinase C) Regulation: feedback inhibited by glucose-6-phosphate
 GTP eventually hydrolyzed to GDP and α-subunit becomes inactive. Examples: cAMP =
glucagon, epinephrine (β, α2); PIP2 = vasopressin, epinephrine (α1)  Glucokinase (hexokinase IV): distribution: liver does not not normally use glucose a
fuel so the only function is storing excess glucose following a meal; β cells of
pancreas uses as a means to measure blood glucose and release insulin accordingly
cGMP signaling: Pathway: hormone binds to receptor on cell membrane (there is also a mutated in the monogenenic, autosomal dominant form of diabetes called Maturity
soluble receptor in the cytoplasm), receptor has intrinsic guanylate cyclase activity (GTP Onset Diabetes of the Young type 2 (MODY2). Kinetics: low affinity → high Km, high
→ cGMP; cGMP activate protein kinase G, protein kinase G mediates smooth muscle capacity → high Vmax. Regulation: induced by insulin (to store glucose in liver after a
relaxation/vasodilation, no G protein required). Examples: ANF → cell membrane meal), no direct feedback inhibition
receptor; NO → diffuses across cell membrane and binds soluble receptor (mechanism
for pharmacologic nitrates (nitroprusside, nitroglycerine))  Phosphofructokinase-1: rate-limiting step; inhibited by ATP, citrate, stimulated by
AMP, fructose-2,6-bisphosphate. Fructose 2,6-bisphosphate synthesized by
Cellular energy production: phosphofructokinase-2

Dietary energy content: Carbohydrate and protein (4 kcal/gm), fat (9 kcal/gm),  Pyruvate kinase: catalyzes substrate-level phosphorylation; inhibited by ATP,
alcohol (7 kcal/gm) alanine, activated by fructose-1,6-bisphosphate

Universal electron acceptors:  Hormonal regulation: fasting state: ↑ glucagon → ↑ cAMP → ↑ protein kinase A → ↑
 Nicotinamides: NAD+; function: catabolic processes (transport reducing equivalents FBPase-2, ↓ PFK-2; fed state: ↑ insulin → ↓ cAMP → ↓ protein kinase A → ↓ FBPase-2,
as NADH) ↑ PFK-2
 NADPH: function: regenerate glutathione (catalyzed by glutathione reductase;
newly reduced glutathione can convert H2O2 to H2O catalyzed by glutathione Disorders of glycolysis:
peroxidase)  Pyruvate kinase deficiency: AR (most commonly); pathophysiology: ↓ ATP
 , biosynthesis (fatty acids, cholesterol, nucleotides), respiratory burst, P-450 generation (inability to maintain Na+/K+ ATPase leads to RBC swelling, RBC lysis)
 Production: see HMP Shunt topic and back up of glycolysis (↑ 2,3-BPG and other glycolytic intermediates).
 Flavin nucleotides: FAD+: function similar to NAD+ (does not carry as much energy; Presentation: chronic hemolysis, ↓ O2 affinity for HbA, due to ↑ 2,3-BPG; no Heinz
generates 2 ATP in ETC while NADH generates 3). Production: succinate bodies unlike glucose 6-phosphate dehydrogenase deficiency.
dehydrogenase (citric acid cycle)
 Hexose monophosphate shunt (HMP): function: generate NADPH required for FA
synthesis, steroid synthesis, reduction of oxidizing agents (H2O2 see figure); provide NADH)
ribose 5-phosphate required for nucleotide synthesis  α-ketoglutarate dehydrogenase complex (requires the same cofactors as the
 Pathway: occurs in cytoplasm of all cells, no ATP consumed or generated. 2 phases: pyruvate dehydrogenase complex: B1, B2, B3, B5, lipoic acid, in
o Oxidative: produces NADPH; glucose 6-phosphate (G6P) → 6-phosphogluconate alcoholics, B1 deficiency leads to Wernicke's encephalopathy (triad of ataxia,
catalyzed by glucose 6-phosphate dehydrogenase (G6PDH): rate limiting step, confusion, ophthalmoplegia); inhibited by ↑ energy (↑ ATP, NADH), inhibited by ↑
activated by NADP+, insulin, inhibited by NADPH; irreversible intermediates in the cycle (↑ succinyl-CoA)
o Nonoxidative: exchanging intermediate substrates between glycolysis and HMP  succinyl-CoA synthetase: generates GTP
shunt catalyzed by transketolase, requires thiamine, reversible  succinate dehydrogenase: a member of both the citric acid cycle and the electron
transport chain
Clinical relevance:  important intermediates:
 Glucose-6-phosphate dehydrogenase (G6PDH) deficiency: pathophysiology: ↓ NADPH  citrate: functions to shuttle acetyl-CoA out of mitochondria for fatty acid synthesis,
production (cells (specifically RBCs) lose protection against oxidizing agents); citrate shuttle
cannot regenerate glutathione; XR; most common human enzyme deficiency, ↑  succinyl-CoA: building block for heme synthesis
prevalence among blacks, ↑ malarial resistance (by shortening the circulation life of  fumarate: enters from the urea cycle
RBCs. Plasmodium does not have enough time for life span, plasmodium does not  malate: functions as a gluconeogenic substrate
have defense against free radicals, ↑ in free radicals kills parasite).  Regulation: energy status control (NO hormonal control)
 Presentation: episodic hemolytic anemia (intravascular hemolysis, normocytic, 2-3
days post precipitating stress: foods (fava beans, common in Mediterranean foods, ETC: function: couple energy stored in electron acceptors (NADH, FADH2) to ATP
presentation: pallor, hemoglobinuria 24-48 post ingestion), drugs (sulfonamides, synthesis; process called oxidative phosphorylation: 3 ATPs per NADH (NADH enters
primaquine, antituberculosis drugs), infection (free radicals generated by the mitochondria from production in cytosol via malate-aspartate/glycerol-3-phosphate
immune system)), Heinz bodies (oxidized hemoglobin that precipitates within shuttle), 2 ATPs per FADH2 (lower energy content than NADH)
RBCs), bite cells (result from the phagocytic removal of Heinz bodies by  Pathway: located in inner mitochondrial membrane; series of carrier enzymes; NADH
macrophages), back pain. and FADH2 create a proton gradient across the inner membrane, pass electrons in a
 Test: active hemolysis screen (Heinz body prep) stepwise fashion, oxygen is the final electron acceptor, flow of proton back down
concentration gradient drives F0F1 ATP synthase complex (net production of ATP)
Pyruvate dehydrogenase complex:  Clinical importance: electron transport inhibitors disrupt membrane bound carrier
 Function: catalyze conversion of pyruvate to acetyl-CoA; net reaction: pyruvate + enzymes, result: ↓ proton gradient, ↓ ATP synthesis, ↓ O2 consumption, ↑
NAD+ + CoA → acetyl-CoA + CO2 + NADH; irreversible intracellular NADH/NAD+ ratio
 Structure: 3 enzymes that require 5 cofactors: thiamine pyrophosphate (B1), FAD (B2),  CO: source: combustion (smoking, fires, car exhaust, grills), paint strippers.
NAD (B3), CoA (B5), lipoic acid; similar to α-ketoglutarate dehydrogenase complex Presentation: ↓ O2 sat, cherry-red lips and cheeks, headache/nausea, tachypnea,
 Regulation: activators of pyruvate dehydrogenase, ↓ in energy status of the cell tachycardia. Treatment: 100% O2
(↑ NAD+/NADH ratio, ↑ ADP), ↑ exercise, ↑ Ca2+, ↑ insulin; inhibited by acetyl-CoA  CN: source: nitroprusside administration byproduct give with thiosulfate to
 Clinical relevance: pyruvate dehydrogenase deficiency: pathophysiology: X-linked consume produced CN, combustion of polyurethane (burning furniture, mattresses),
(variable penetrance in females) accumulation of pyruvate, alanine; lactic acidosis mining (gold), metal extraction. Presentation: seizures, tachypnea, tachycardia,
(pyruvate shunted to lactate to regenerate NAD+). Presentation: neurological headache, flushing. Treatment: sodium thiosulfate (forms thiocyanate, less-toxic
defects. Treatment: ↑ ketogenic nutrients (lysine and leucine) metabolite, renally excreted), nitrites (convert hemoglobin to methemoglobin
 Arsenic poisoning: inhibits pyruvate dehydrogenase (via inhibition of lipoic acid); (ferrous to ferric), does not allow cyanide transport to mitochondria, must be given
presentation: vomiting, rice water stools, garlic breath shortly after exposure)

Citric acid cycle: Krebs cycle, tricarboxylic acid (TCA) cycle Note: victims of house fires may have both CO and CN poisoning
 Function: generate high amounts of NADH/FADH2 that act as fuel for ATP synthesis in
the electron transport chain  ATPase inhibitors: directly inhibit mitochondrial ATP synthase. Result: ↑ proton
 Pathway: occurs in the mitochondria, requires O2 to function, net equation: acetyl-CoA gradient; no ATP is produced because electron transport stops e.g. oligomycin
+ 3 NAD+ + FAD + GDP + Pi → 2CO2 + 3 NADH + FADH2 + GTP + CoA. Will  Uncoupling agents: "uncouples" ATP production from the proton gradient, ↑
theoretically yield 12 ATP (if ETC were 100% efficient), both carbons of acetyl-CoA permeability of membrane. Result: ↓ proton gradient, ↑ O2, NADH consumption, ATP
leave as CO2 in two of the reactions, conversion of isocitrate to alpha ketoglutarate, synthesis stops, but electron transport continues produces heat. Examples: 2,4-DNP,
conversion of alpha ketoglutarate to succinyl-CoA. aspirin/salicylates (fevers often occur after aspirin overdose), thermogenin in
brown fat (UCP protein, generates heat for newborns)
Important enzymes:  CO (carbon monoxide) and CN (cyanide): site of inhibition: cytochrome a/a3
 isocitrate dehydrogenase (stimulated by ↓ energy, ↑ ADP; inhibited by ↑ energy, ATP,  Antimycin: SOI: sytochrome b/c1
 Doxorubicin: SOI: CoQ o Other enzymes: lactate dehydrogenase: lactate → pyruvate, requires free NAD+
 Retenone (pesticide): SOI: NADH dehydrogenase  Regulation: stimulation (glucagon, acetyl CoA, citrate), inhibition (high NADH/NAD+
ratio; alcohol may cause elevated NADH/NAD+ ratio leading to hypoglycemia)
Glucose transport:
 Sodium/glucose cotransporter (SGLT): function: transport glucose actively across Glycogen: Structure: polymer of glucose: straight chain with α-1,4-bond, branches with
lumen against concentration gradient (energy provided by transport of sodium α-1,6-bond. Glycogen granule: core: glycogenin
down its concentration gradient). Location: small intestine (SGLT1; 2:1 Na+:Glu),  Function: energy reserves that can provide glucose during a fast or ↑ energy
proximal tubule of nephron (SGLT2; 1:1 Na+:Glu) demand (supplies exhausted in < 24 hrs), stored mainly in the liver and muscle.
Muscle does not have glucose-6-phosphatase so it cannot release free glucose;
 GLUT-1: function: basal glucose uptake (high affinity: transporters saturated at stores for its own consumption, liver does have glucose-6-phosphatase so it can
normal blood glucose levels, ensures glucose entry to cells). Location: wide release free glucose and can use supplies to maintain blood glucose levels.
distribution in tissues in the body (brain, erythrocytes, endothelial cells, cornea etc.)  Glycogenesis: Glycogen synthesis
 Pathway: glucose-6-phosphate converted to glucose-1-phosphate, UDP group added
 GLUT-2: function: low affinity glucose uptake (in the fasting state glucose does not to form UDP-glucose, UDP-glucose added to polymer in an α-1,4 linkage catalyzed
enter cells, mediates glucose surplus storage in liver when blood glucose levels rise, by glycogen synthase (rate limiting step of glycogen synthesis), polymer rearranged
facilitates insulin release in β-cells). Location: hepatocytes, pancreatic β-cells, to create α-1,6 linked branches (catalyzed by branching enzyme, deficiency =
kidney, small intestines Anderson disease)
 Regulation: glycogen synthase (in liver: activated by insulin, inhibited by glucagon,
 GLUT-3: function: high affinity glucose uptake (glucose preferentially accessed by epinephrine; in muscle: activated by insulin, inhibited by epinephrine)
neurons in low-glucose states). Location: brain, neurons
Glycogenolysis: Glycogen catabolism
 GLUT-4: function: insulin-controlled uptake of glucose, basal level of glucose intake  Pathway: glucose-glucose bond broken by addition of a phosphate catalyzed by
without insulin (presence of insulin ↑ translocation of transporters to the cell glycogen phosphorylase (rate limiting step of glycogenolysis; hepatic deficiency =
membrane, ↑↑↑ glucose uptake, also stimulated by exercise). Location: adipocytes, Hers disease (type VI); muscle deficiency = McArdle disease (type V)), glucose-1-
myocytes, cardiomyocytes phosphate freed (converted to glucose-6-phosphate; debranching enzymes removes
α-1,6 linked branches; deficiency = Cori's disease (type III)), liver converts glucose-
Gluconeogenesis: 6-phosphate to glucose catalyzed by glucose-6-phosphatase (deficiency = von
 Function: de novo glucose synthesis: effectively glycolysis in reverse, can maintain Gierke disease (type I)), muscle puts glucose-6-phosphate into glycolysis
blood glucose when glycogen stores are exhausted, must supply brain and RBCs  Regulation: glycogen phosphorylase in liver (activated by epinephrine/glucagon via
which utilize glucose for energy; is NOT a source of energy for the liver (hepatocytes cAMP/protein kinase A, inhibited by insulin; remember: exact opposite of glycogen
use β-oxidation to supply the energy needed for gluconeogenesis). Potential synthase: hepatic glycogen regulatory processes both turn on forward direction and
substrates: all amino acids (except for leucine and lysine), lactate (produced in turn off reverse). In skeletal muscle: activated by epinephrine, AMP, Ca2+, inhibited
anaerobic glycolysis), glycerol-3-phosphate (produced in fat catabolism), propionyl- by insulin, ATP), remember: since muscular glycogen can only supply itself, it is
CoA, produced in odd-carbon fatty acid catabolism regulated by its own energy supply (AMP/ATP ratio); while liver must supply
 Pathway: location: hepatocytes (primary), kidney, enterocytes, NOT muscle (no energy to many other tissues, it functions independently of AMP/ATP ratio in
glucose-6-phosphatase, cannot release free glucose) hepatocytes
 Enzymes: involves both mitochondrial and cytosolic enzymes, several steps of
glycolysis are reversible, the non-reversible steps must be bypassed with special Glycogen storage disease (glycogenolyses): all disorders have abnormal glycogen
gluconeogenic enzymes: metabolism, leads to an accumulation of glycogen within cells, organ dysfunction.
o Pyruvate carboxylase: pyruvate → oxaloacetate; requires biotin and ATP; Remember: disorders numbered in order of pathway from end (glucose release) to
activated by acetyl-CoA; oxaloacetate must be converted to malate to exit the beginning (breakdown of glycogen polymer)
mitochondria via the malate-aspartate shuttle in mitochondria  Glucose release: Type I: von Gierke: lacks glucose-6-phosphatase. Presentation: liver
o PEP carboxykinase (PEPCK): oxaloacetate → phosphoenolpyruvate (PEP); cannot release stored glucose: hepatomegaly, severe hypoglycemia. Body must rely
requires GTP; activated by glucagon and cortisol in both cytosol and on fat/protein catabolism for energy: hyperlipidemia, hyperuricemia, lactic acidosis,
mitochondria normal glycogen structure. Tests: stimulation test with glucagon, fructose, galactose:
o Fructose-1,6-bisphosphatase: fructose-1,6-bisphosphate → fructose-6-P; does not ↑ serum glucose
important control point of gluconeogenesis; activated by ATP, inhibited by AMP  Lysosomal pathway: Type II: Pompe "trashes the Pump (heart)": lacks lysosomal α1,4-
and fructose-2,6-bisphosphate in cytosol glucosidase, degrades glycogen-resembling material in endosomes. Presentation:
o Glucose-6-phosphatase (G6P): glucose-6-P → glucose in ER of hepatocytes. buildup of glycogen in cardiac muscle; electron dense granules inside lysosomes,
Clinical relevance: von Gierke disease = G6P deficiency cardiomegaly, hypertrophic cardiomyopathy
 Branching/debranching: episodes of vomiting after feeding ever since birth though this has not previously been
o Type III: Cori: lacks debranching enzyme. Remember: Cori = can't Catabolize extensively investigated. Physical exam shows the development of bilateral cataracts so
branches, 6-pack core - alpha 1,6 glucosidase defective. Presentation: liver cannot based on clinical suspicion a metabolic panel is conducted in order to evaluate for
break down glycogen past a branch point: hepatomegaly, hypoglycemia, abnormal disorders of galactose metabolism.
glycogen structure (short outer glycogen chains)  Source: lactose in dairy products: disaccharide composed of glucose and galactose;
o Type IV: Anderson: lacks branching enzyme; remember: Anderson = can't Add hydrolyzed by lactase in brush border of small intestine
branches. Presentation: liver cannot form branched glycogen granules: hypotonia,  Metabolic pathway: trapped in cell by galactokinase; converted into glucose-1-
cirrhosis phosphate through UDP mediated epimerization, glucose-1-phosphate can then be
 Phosphorylase: used: directly in glycogenesis, converted to glucose-6-phosphate and used in
o Type V: McArdles: lacks muscle phosphorylase; remember: McArdles = Muscle, glycolysis or released as free glucose in hepatocytes
can't breakdown glycogen to glucose-1-phosphate. Presentation: muscle  Pathophysiology:
weakness/cramps upon exertion, myoglobinuria, normal glycogen structure o Galactokinase deficiency: presentation: galactosemia/galactosuria, cataracts in
o Type VI: Hers: lacks hepatic phosphorylase; remember: Hers = Hepatic. childhood, excess galactose is converted to galactitol, catalyzed by aldose
Presentation: hepatomegaly, fasting hypoglycemia; can be mild due to reductase, galactitol is osmotically active. Treatment: galactose free diet
gluconeogenic compensation. o Gal-1-P uridyl transferase deficiency: similar to galactokinase deficiency but
more severe, Gal-1-P acts as a phosphate sink; presentation: early cataracts,
Sorbitol: A 69-year-old man presents to your clinic with a chief complaint of changes in more severe, with vomiting/diarrhea after milk ingestion, liver disease, lethargy,
his vision. He has a past medical history of diabetes mellitus type II and medication non- mental retardation, hepatomegaly, Hyperbilirubinemia, jaundice.
compliance. The patient states that after a very large meal in which he consumed a Treatment: galactose free diet
family-sized bucket of chicken, a glass of maple syrup, and an entire wedding cake he
noticed that his vision became very blurry. In fact, he notes this happens every time he Lactose intolerance: mechanism: lactase deficiency. Primary: hereditary; ↑ frequency in
eats a large meal (sorbitol accumulation in the lens & lens swelling). african americans and Asians; age-dependent. Secondary: post-gastroenteritis.
 Presentation: bloating, cramps, osmotic diarrhea; symptoms due to fermentation of
 Pathway: alternative method of trapping glucose in the cell; tissues with sorbitol indigestible lactose by intestinal bacteria, leads to production of H2, CH4, organic
dehydrogenase: liver, ovaries, seminal vesicles, lens (at low level of activity), tissues acids (H2 used in diagnosis, detected on breath following oral lactose load)
without sorbitol dehydrogenase: Schwann cells, retina, kidney. Note: galactose can  Treatment: eliminate dairy from diet, take lactase pills when consuming dairy
also be converted to an aldose
 Clinical relevance: prolonged hyperglycemia (commonly caused by uncontrolled Fat Metabolism:
diabetes mellitus); pathophysiology: glucose enters cells and is converted to sorbitol  Fatty acid metabolism: Structure: long chain of carbons with carboxyl group on one
in tissues without sorbitol dehydrogenase or low levels of activity (sorbitol trapped end, can have a variable amount of double bonds (double bonds make a fat
in cell and is osmotically active). Presentation: pathology directly linked to which unsaturated, naturally in a cis configuration: trans fats are unnatural and created via
tissues have aldose reductase but lack sorbitol dehydrogenase: peripheral hydrogenation of vegetable oils (↑ risk of atherosclerosis), double bonds ↓ melting
neuropathy, cataracts, retinopathy (all symptoms of chronic diabetes) temperature: plant fat (e.g. olive oil) is unsaturated and liquid at room temperature,
animal fat (eg. butter) is saturated and solid at room temperature)
Fructose metabolism: source: sucrose in fruits and sweeteners (disaccharide of glucose  Nomenclature:
and fructose hydrolyzed by sucrase in brush border of small intestine). o e.g. palmitic acid: C16:0; 16 carbons with no double bonds, numbered with
 Pathway: liver is main site of metabolism, also in renal proximal tubule; DHAP and carboxyl carbon as 1
glyceraldehyde enter into glycolysis downstream of regulation o e.g. linoleic acid: C18:2 (9,12); 18 carbons with 2 double bonds (one at the 9th
 Clinical relevance: and one at the 12th carbon)
 Fructokinase deficiency ; benign; presentation: fructosuria (does not have the osmotic o omega system: count opposite to the numbered system (i.e. carboxyl carbon is
pathologies (e.g. cataracts) associated with galactokinase deficiencies (fructose is counted last), used to number unsaturated fats e.g. linoleic acid: omega 6 family,
not an aldose and therefore not substrate for aldose reductase). double bond at position "12" is 6 in from the opposite side (18 carbons in total)
 Fructose 1-P aldolase (aldolase B) deficiency: aka hereditary fructose intolerance; AR;  Essential fatty acids (FA): cannot be synthesized. Examples:
more severe than fructokinase deficiency because fructose 1-P acts as a phosphate o Linoleicacid: omega 6, can be used as a precursor for arachidonic acid (becomes
sink; presentation: fructosuria, lethargy, hypoglycemia, liver and proximal renal an essential fatty acid if linoleic acid is absent)
tubule disorder (Hyperbilirubinemia, hyperuricemia (degradation of ADP due to o linolenic acid: omega 3, ↓ risk of CV disease. Remember: omega 3 saves you from
loss of Pi)); treatment: fructose (and sucrose)-free diet. triple bypass (found in cold water fish, nuts)

Galactose metabolism: A 2-week-old boy is brought to the emergency department after Fatty acid synthesis: FA synthesis:
progressive lethargy over the course of the previous week. His parents report extended
o pyruvate (carbohydrate) → acetyl-CoA: activated by insulin, functions to store
excess carbs as fat, occurs in the mitochondria via pyruvate dehydrogenase Ketone bodies:
o acetyl-CoA + oxaloacetate → citrate: shuttled out of mitochondria into Structure: two types: acetoacetate, β-hydroxybutyrate:
cytoplasm (citrate shuttle), split back to acetyl-CoA and oxaloacetate  β-hydroxybutyrate + NAD+ → acetoacetate + NADH
o acetyl-CoA + CO2→ malonyl-CoA: catalyzed by acetyl-CoA carboxylase, biotin  ↑ NADH:NAD+ ratio results in ↑ β-hydroxybutyrate:acetoacetate ratio
required, activated by insulin o 1 ketone body = 2 acetyl-CoA
o malonyl-CoA → CO2 + 2 carbons on fatty chain: catalyzed by FA synthase Function
 requires NADPH o produced by the liver: brain can use ketones if glucose supplies fall , >1 week of
 humans make palmitic acid (16:0) as stored fat: only de novo fat possible fasting
 for 1 palmitic acid requires: 8 acetyl-CoA, 7 ATP, 14 NADPH o can provide energy to body in prolonged energy needs: prolonged starvation,
glycogen and gluconeogenic substrates are exhausted
Fatty acid catabolism: Break down via β-oxidation: occurs in hepatocytes, myocytes, o can provide energy if citric acid cycle unable to function
adipocytes (neurons cannot use fat as energy: FAs do not cross BBB). Pathway location  diabetic ketoacidosis: cycle component (oxaloacetate) consumed for
differs based on length of Fas: short/medium (2-12 carbons) diffuse in mitochondria, gluconeogenesis
long (14-20 carbons) utilizes carnitine shuttle:  alcoholism: ethanol dehydrogenase consumes NAD+ (converts to NADH)
▪ carnitine added to FA in the intermembrane space of the mitochondria catalyzed  ↑ NADH:NAD+ ratio in liver favors use of oxaloacetate for ketogenesis
by carnitine acyltransferase (CAT) -1, inhibited by malonyl-CoA so as to prevent rather than gluconeogenesis.
newly synthesized FAs from being degraded o RBCs cannot use ketones as they lack mitochondria
▪ carnitine: FA transported into the matrix catalyzed by the carnitine transporter Synthesis: occurs in hepatocyte mitochondria (liver cannot use ketones as energy, lacks
▪ carnitine exchanged for CoA catalyzed by carnitine acyltransferase (CAT)-2 β-ketoacyl-CoA transferase (thiophorase) which converts acetoacetate to acetoacetyl)
▪ clinical importance: myopathic CAT deficiency: presentation: myoglobinuria o under normal conditions acetoacetate = β-hydroxybutyrate
(muscle aches/weakness), ↑ TG content in muscles (unable to use as energy), o HMG CoA synthase is rate limiting enzyme
provoked by prolonged use of muscle; very long (>20 carbons) oxidized in Clinical relevance:
peroxisome Ketoacidosis: pathogenesis: ↑ ketone levels caused by poorly controlled type I diabetes
mellitus (liver ketone production exceeds ketone consumption in periphery), possible in
β-oxidation pathway: occurs in the mitochondrial matrix, reverses FA synthesis: type II diabetes mellitus but rare, alcoholism (chronic hypoglycemia results in ↑ ketone
Removing an acetyl-CoA and producing NADH and FADH2 catalyzed by fatty acyl-CoA production). Presentation: β-hydroxybutyrate > acetoacetate (due to ↑ NADH:NAD+
dehydrogenase. Two types: long-chain acyl-CoA dehydrogenase (LCAD), medium-chain ratio), acetone gives breath a fruity odor, polyuria, ↑ thirst.
acyl-CoA dehydrogenase (MCAD); blocked by ackee fruit toxin. Tests: ↓ plasma HCO3, hypokalemia (individuals are initially hyperkalemic (lack of insulin
 Creates most of the energy used by the liver: acetyl-CoA created in liver does not enter + acidosis) because K leaves the cells, overall though the total body K is depleted, replete
the citric acid cycle K in these patients once the hyperkalemia begins to correct), nitroprusside urine test for
 Clinical importance: MCAD deficiency: presentation: non-ketotic hypoglycemia, C8- ketones may not be strongly + (does not detect β-hydroxybutyrate, state favored by ↑
C10 acyl carnitines in the blood (liver unable to break FAs down further than C8- NADH:NAD+ ratio, should use a test specific for β-hydroxybutyrate)
C10), no ketone bodies (liver unable to produce ketones from β-oxidation), fasting
hypoglycemia (liver unable to produce enough energy from β-oxidation to supply triglycerides:
gluconeogenesis); symptoms often precipitated by infection or stress. Treatment:  Function: storage form of fatty acids in adipose tissue
low fat diet with frequent meals of high carbs.  Synthesis: 3 fatty acids + glycerol-3-phosphate → triglyceride. Glycerol-3-phosphate
provided by: reduction of DHAP from glycolysis (by adipocytes and hepatocytes),
Propionic acid pathway: Production: β-oxidation of odd-numbered fatty acids (2 carbon phosphorylation of free glycerol by glycerol kinase (only hepatocytes)
acetyl-CoA groups removed from chain until there are 5 carbons remaining, 5 carbon  Catabolism :
chain split into 1 acetyl-CoA (2C) + 1 propionyl-CoA (3C)) o triglyceride hydrolyzed into 3 fatty acids + glycerol, catalyzed by hormone
 Pathway: propionyl-CoA → methylmalonyl-CoA catalyzed by propionyl-CoA sensitive lipase, ↑ regulated by epinephrine, cortisol , ↓ regulated by insulin
carboxylase (requires biotin (B7)), methylmalonyl-CoA → succinyl-CoA catalyzed by o glycerol is a gluconeogenic substrate in the liver: converted to DHAP
methylmalonyl-CoA (requires B12; deficiency of B12 results in a blockage of this step, o fatty acids undergo β-oxidation in liver and tissues: can travel in serum
result is ↑↑↑ methylmalonate, can cause irreversible neuropathy due to pathologic complexed with albumin
synthesis of myelin with methylmalonate, a means to determine whether a patient
with megaloblastic anemia is deficient in folate or B12: methylmalonic aciduria not Cholesterol: Function: component of cell membrane, precursor for hormone synthesis
seen in folate deficiencies), succinyl-CoA → citric acid cycle (can be converted to (steroids, vitamin D), precursor for bile acid synthesis (means of excretion; >95% of bile
malate: gluconeogenic: exception to rule that fatty acids can be converted to acids are reabsorbed; ↑ in bile acid production used as a means to treat
glucose). hypercholesterolemia), cholestyramine (resin that binds bile acids in the GI tract and
prevents reabsorption, increases serum cholesterol usage in bile acid production, ↑ receptor on liver and tissues).
regulates LDL receptor)
 Sources: dietary intake (circulating serum LDL: uptake via endocytosis of LDL:LDL VLDL: Function: transports TGs from liver → tissues. Structure: apoB-100, form
receptor complex; circulating serum HDL: transfer of cholesterol from HDL to packaged and secreted from liver; apoC-II activates lipoprotein lipase (catalyzes
hepatocyte via scavenger receptor (SR-B1), high levels of this receptor in hydrolysis of triglycerides into individual FAs for absorbtion; ↑ regulated by insulin);
hepatocytes and steroid producing tissues), de novo synthesis (occurs in apoE mediates uptake by the liver
hepatocytes; HMG-CoA reductase is rate limiting enzyme, inhibited by statins,
glucagon, cholesterol, activated by insulin) Chylomicrons: Function: transport cholesterol, TGs, FAs, and fat soluble vitamins
 Regulation: ↑ in intrahepatic [cholesterol] ↓ expression of: HMG-CoA reductase (↓ de (intestine → tissues), released from intestinal lumen cells into lymphatics. Structure:
novo cholesterol synthesis), LDL-receptor (↓ in cholesterol-containing LDL uptake apo-48 (form packaged and secreted by intestine), apoC-II (activates lipoprotein lipase,
from serum), scavenger receptor (SR-B1) (↓ cholesterol uptake from HDL) added from HDL), apoE (mediates uptake by the liver, added from HDL)
 Transport: cholesterol is fat-soluble: ↑ transport by synthesis of a cholesteryl ester
(dissolves into center of HDL, catalyzed by lecithin-cholesterol acyltransferase lipoprotein disorder: A 12-year-old boy presents to the emergency department with
(LCAT): enzyme in serum activated by apoA-1 of HDL) chest pain after gym class. His parents note that he has been having increased episodes
 Pathology: of difficulty catching his breath after exertion and has had previous episodes of chest
o familial hypercholesterolemia: defective LDL receptor; AD; presentation: pain on exertion. Upon physical exam yellow deposits are found on his heels and on his
cholesterol deposition in skin, xanthelasma (in eyelid), tendon xanthomas (in skin eyebrows. Based on clinical suspicion a LDL level is obtained and is found to be 480
above tendons), ↑ risk for coronary heart disease mg/dL. He is referred to a geneticist for evaluation of deficiencies in LDL receptors and is
o atherosclerosis: risk factors (causes of endothelial cell damage): smoking, ↑ LDL prescribed a statin.
in serum, homocystinemia, diabetes; protective factors: antioxidants (vitamin E,
protects LDL from oxidation); process: injury to endothelial cells of blood vessels, Hyperlipoproteinemia:
an inflammatory state is induced (T-cell activation, similar to a granuloma),  Type I hyperlipoproteinemia (hyperchylomicronemia): pathophysiology: deficiency in
cholesterol in the blood deposits in blood vessels, LDL oxidized and phagocytosed lipoprotein lipase or apoC-II results in ↑↑↑↑ TGs, chylomicrons. Presentation:
by macrophages (can become full of cholesterol: foam cells), fatty streak formed, eruptive xanthomas, steatosis, abdominal pain post-fat ingestion, retinitis
fatty streak enlarges, fibrous cap formed (smooth muscle and endothelial cells pigmentosa
migrate over the fatty streak, underlying tissue forms necrotic core), cap at risk of  Type II hyperlipoproteinemia: pathophysiology: deficiency in LDL receptors
rupturing (exposes underlying endothelium and causes thrombosis)  type IIa (familial hypercholesterolemia) results in ↑↑↑↑ LDL (>260 mg/dL), type IIb
(familial combined hyperlipidemia) results in ↑↑↑↑ LDL, TGs, cholesterol
Apolipoprotein: A 50-year-old woman presents to her primary care physician for a  Presentation: deposition of cholesterol in normal tissue (xanthomas, xanthelasma),
regular checkup. She states that for the past four months she has not experienced ↑↑ risk for coronary heart disease
menses. Her primary care physician suspects that she has gone through menopause. On  Type III hyperlipoproteinemia (familial dysbetalipoproteinemia): pathophysiology:
routine lab work, her LDL has increased as compared to one year ago, and her HDL has deficiency in apolipoprotein E. Remember: III lacks E; results in ↑↑↑ in remnants
decreased drastically: (chylomicron/ IDL), apolipoprotein normally clears remnants (empties)
 Presentation: similar to type II hyperlipoproteinemia, ↑↑ risk for coronary heart
Mechanisms for fatty acid and cholesterol transport: different types that vary by: disease
Density: HDL (high density), LDL (low density), IDL (intermediate density), VLDL (very  Type IV hyperlipoproteinemia (familial hypertriglyceridemia): pathophysiology: ↓
low density), chylomicrons; arranged from most to least dense. types of apolipoprotein: removal or ↑ production of VLDL results in ↑↑↑ in VLDL. Presentation: pancreatitis
 Type V hyperlipoproteinemia: type I + type IV; ↓ lipoprotein lipase + ↑ VLDL.
HDL: Function: transfer cholesterol (tissues → liver), good cholesterol. Structure : apoA-1 Remember: 1+4=5
(activates lecithin cholesterol acyltransferase (LCAT)), apoE/apoC-II (apolipoproteins  Acquired hypercholesterolemia: oral contraceptive, obstructive jaundice
donated to chylomicrons and VLDL)  Acquired hypertriglyceridemia: alcoholism, renal failure, diabetes mellitus
 Treatment for hyperlipoproteinemias: dietary modifications (type I), statins (type II -
LDL: Function: transfer cholesterol from liver → tissues (most recycles back to be IV), niacin (type II, IV, V), fibrates (type IIa, IV, V), bile acid sequestrants (type IIa)
absorbed by the liver); bad cholesterol. Structure: apoB-100; mediates endocytosis of
LDL by binding to apoB-100 (LDL) receptor on liver and tissues Hypolipoproteinemia: Abetalipoproteinemia: AR; pathophysiology: deficiency in
apolipoprotein B-48 and B-100 (remember: A (without) beta (B)), ↓ chylomicrons (B-
IDL: Function: VLDL + TGs → IDL; picks up cholesterol esters from HDL, catalyzed 48), VLDL/LDL (B-100). Presentation: malabsorption of fat (can enter enterocytes but
by cholesterol ester transfer protein (CETP), IDL + cholesterol from HDL → LDL. cannot exit because it cannot be packaged for release in lipoproteins, leads to histological
Structure: apoE (recieves from HDL, mediates uptake by the liver, remember: apoE appearance of fat droplets inside enterocytes), ↓ vitamin E absorption, ataxia, hemolytic
empties to liver), apoB-100 (mediates endocytosis of LDL by binding to apoB-100 (LDL) anemia with acanthocytes
High yield:  fatty acid synthesis
▪ Remember that type I, IIa, and IV are the most common types of  glucose uptake in fat, muscle, liver
hyperlipoproteinemia  protein anabolism
 I and IV present with pancreatitis, IIa presents with early symptoms of ACS  insulin-resistant tissues
 Treat all types other than I with statins, niacin, and fibrates  brain
 RBCs
Metabolism overview: Rate limiting enzymes:  Post-absorptive state
Process Enzyme o between meals
Glycolysis Phosphofructokinase-1 (PFK-1) o epinephrine/glucagon in control
 glucagon-sensitive tissues
Gluconeogenesis Fructose-1,6-bisphosphatase  adipocytes
TCA cycle Isocitrate dehydrogenase  hepatocytes
Glycogen synthesis Glycogen synthase  pathways favored by glucagon
Glycogenolysis Glycogen phosphorylase  glycogenolysis
 fatty acid catabolism
HMP shunt Glucose-6-phosphate dehydrogenase (G6PD)
 protein catabolism
De novo pyrimidine  gluconeogenesis
Carbamoyl phosphate synthetase II
synthesis  glucagon-resistant tissues
De novo purine synthesis Glutamine-PRPP amidotransferase  brain
Urea cycle Carbamoyl phosphate synthetase I  RBCs
Fatty acid synthesis Acetyl-CoA carboxylase (ACC)  Starvation
o days 1-3
Fatty acid oxidation Carnitine acyltransferase I  main source of energy for body
Ketogenesis HMG-CoA synthetase  catabolism of triglycerides in fat stores
Cholesterol synthesis HMG-CoA reductase  main source of energy for brain
 glucose from gluconeogenic conversion of lactate and alanine
Metabolism of exercise and starvation state: o > 3 days
 100 meter sprint  main source of energy for body
o energy source used  catabolism of triglycerides in fat stores
 pre-synthesized ATP  main source of energy for brain
 creatine phosphate  mostly glucose with some ketone bodies
 anaerobic glycolysis o > several weeks
 100 meters to 1 kilometer  main source of energy
o energy source used  degradation of muscle and organs
 oxidative phosphorylation  after adipose tissue store have been exhausted
 main source of energy for brain
 1 kilometer to a marathon
o energy source used  2/3 ketone bodies
 glycogen  1/3 glucose
note: RBCs are always dependent on glucose
 β-oxidation of fatty acids
Pyruvate metabolism: A 23-year-old man is running a marathon. In the last mile of the
 Absorptive state race he experiences an intense burning in his muscles. Once he completes the race and
o immediately after eating rests, the burning slowly subsides. He asks his physician the next day why this may have
o insulin in control occurred and what he can do to mitigate it. His physician explains that a training
 insulin-sensitive tissues regimen can increase his ability to perform aerobic metabolism, and thus decrease the
 adipocytes conversion of pyruvate to lactate (via lactate dehydrogenase) which caused the burning
 myocytes in his muscles that he experienced.
 hepatocytes Pathways
 pathways favored by insulin  pyruvate → lactate
 glycogenesis o catalyzed by
 lactate dehydrogenase (LDH)  hypoxia alone cannot explain this effect
o reversible  generates high amounts of biosynthetic substrates for growth
o generated in anaerobic glycolysis  unique protein expression profile
 allows conversion of NADH → NAD+  Metabolic changes
o in the liver, LDH converts lactate to pyruvate o transport of glucose into cells
 for gluconeogenesis or for metabolism to acetyl-CoA  ↑↑ expression of GLUT1
 Cori cycle  allows for ↑↑ glucose uptake
 shifts energy generation from periphery to the liver  ↑↑ expression of hexokinase 2
 pyruvate → acetyl-CoA  promotes glycolysis
o catalyzed by  inhibits apoptosis
 pyruvate dehydrogenase (PDH)  ↑↑ affinity for ATP
o irreversible  ↓ inhibition by G6P
o acetyl-CoA enters the citric cycle o glycolysis
 pyruvate → oxaloacetate  ↑↑ expression of fetal pyruvate kinase
o catalyzed by  favors the anaerobic pathway
 pyruvate carboxylase (PC)  ↑↑ shunting into pentose phosphate shunt
o irreversible  NADPH needed for biosynthesis
o oxaloacetate can o fatty acid metabolism
 replenish the citric acid cycle  ↓↓ β-oxidation
 substrate for gluconeogenesis  ↑↑ lipid biosynthesis
 pyruvate → alanine o amino acid metabolism
o catalyzed by  ↑↑ utilization of glutamine
 alanine transaminase (ALT)  replenish intermediates in the TCA cycle
o reversible  e.g. oxaloacetate
o alanine carries amino groups to the liver from muscle o cell growth
o in the liver, ALT converts alanine to pyruvate  ↑↑ expression of Akt
 for gluconeogenesis  controls proliferation of cells
 regulates growth downstream of insulin
Ethanol metabolism: Function: elimate ethanol (EtOH): CNS depressant/toxin  Clinical importance
Kinetics: NAD+ is limiting reagent, alcohol dehydrogenase operates via zero-order o cancer therapy
kinetics, inhibitors (fomepizole inhibits alcohol dehydrogenase, antidote for suspected  inhibitors of pyruvate kinase
ethylene glycol or methanol poisoning; disulfiram (Antabuse) inhibits acetaldehyde  forcing the cancer cell to use the TCA cycle
dehydrogenase, acetaldehyde accumulates, leads to hangover symptoms, prescribed to
 inhibitors of fetal pyruvate kinase
help recovering alcoholics)
 ↑ amount of adult pyruvate kinase
Clinical relevance: ethanol hypoglycemia: pathophysiology: ↑ in EtOH metabolism → ↑
 ↑ use of the TCA cycle
NADH/NAD+ ratio, NADH/NAD+ ratio changes energy generating kinetics, lactate
DNA:
favored over pyruvate (lactate + NAD+ → pyruvate + NADH; no free NAD+ for required
Nucleus:
conversion), glycerol-3-phosphate favored over DHAP (G3P + NAD+ → DHAP + NADH),
Nuclear envelope: membrane bilayer structure, continuous with ER, contains nuclear
malate favored over oxaloacetate (malate + NAD+ → OAA + NADH). Presentation: lactic
pores: controls transport between cytoplasm and nucleus, more active cells have more
acidosis (↑ lactate), fatty liver (↑ G3P results in ↑ synthesis of TGs), ethanol + extreme
nuclear pores, small molecules freely traffic, large molecules require active transport
physical exertion (severe hypoglycemia, EtOH inhibits the Cori cycle by consumption of
(mediated by importins and exportins, nuclear localization signal (NLS) provides signal
free NAD+: conversion of lactate to glucose in anaerobic metabolism, lactic acidosis).
for nuclear access)
Nuclear lamina: on inner face of nuclear envelope, fibrous network of proteins, role:
Warburg hypothesis:
attach to chromatin, participate in construction and deconstruction of nucleus during cell
 Principle idea division (phosphorylation by lamin kinase in prophase results in blebbing of nuclear
o cancer cells generate energy via a unique mechanism envelope)
 anaerobic glycolysis
 low energy generating pathway Nucleolus: location of rRNA synthesis, site of ribosomal assembly
 cancer cells need high energy to facilitate their rapid division
Nucleic acid structure:  Clinical importance
 Nitrogenous bases o anticancer drugs
o purines  can intercalate DNA
 structure  daunorubicin, doxorubicin
 2 rings  can bind DNA
 examples  cisplatin
 adenine (A), guanine (G) Chromatin structure:
 found in DNA and RNA Composition:
 xanthine, hypoxanthine, uric acid DNA: - charge
 not found in either DNA or RNA Histone proteins: + charge from Lys and Arg residues; H2A, H2B, H3, H4 (core proteins);
o pyrimidines H1 (linking protein); post-translational modification of histone tails, forms a code,
 structure interpreted by effector proteins to regulate transcription downstream. Histone
 1 ring acetyltransferase (HAT): acetyl group added which blocks positive charge of histone
 examples protein and loosens interaction with DNA, ↑ transcription. Histone deacetylase (HDAC):
 cytosine (C) removes acetyl group which exposes positive charge and tightens interactions, ↓
 found in DNA and RNA transcription
 thymine (T) Electrostatic attraction of DNA with histone proteins
 found in DNA
Organization:
 uracil (U)
10 nm chromatin: DNA wraps around dimer of H2A:H2B:H3:H4; called nucleosome;
 found in RNA
sensitive to nuclease activity
 Nucleoside 30 nm chromatin: nucleosomes held together by H1; not sensitive to nuclease activity
o nitrogenous base + ribose 30 nm fiber loops: further condensation
 Nucelotide
o nitrogenous base + ribose + phosphate Euchromatic/heterochromatic: euchromatic = accessible to transcription,10 nm through
 3'-5' phosphodiester bond connects ribose and phosphate 30 nm fiber loops; heterochromatic = not accessible to transcription, any greater
 DNA condensation than 30 nm fiber loops, condensed to save room
o nucleotide interactions
 H-bonding De novo nucleotide synthesis:
 G with C
 3 H-bonds Phosphoribosyl pyrophosphate (PRPP): sugar building block formed in nucleotide
 stronger synthesis, added to nitrogenous base to form nucleoside, formed from ribose-5-
 A with T phosphate (product of pentose phosphate shunt: ATP + ribose-5-phosphate = PRPP +
 2 H-bonds AMP; catalyzed by PRPP synthetase). Once PRPP is made it can add to either a de novo or
 weaker salvaged base
 A with U in RNA
 melting or denaturing Pyrimidine: pathway diagram, important enzymes:
 H-bonds disrupted with changes in temperature, pH, chemical carbamoyl phosphate synthetase-2: rate limiting step; not the same carbamoyl
agents phosphate as in urea cycle
ribonucleotide reductase: inhibited by hydroxyurea; also reduces UDP, CDP, ADP, GDP;
 ↑ GC content = ↑ melting temperature
dADP and dATP negatively feedback and inhibit enzyme. Result = ↓ dTMP, dUDP, dCDP,
 DNA can form correct structure again if disrupting agent is
dADP, dGDP. Note: good to target thymidine synthesis because it is not involved in RNA
removed slowly (reanealing)
thymidylate synthase: inhibited by 5-fluorouracil (5-FU); result = ↓ dTMP
 key principle of Southern blotting and PCR
dihydrofolate reductase: inhibited by: methotrexate (MTX) in eukaryotes, trimethoprim
 see Biological lab techniques section (TMP) in prokaryotes (sulfamethoxazole (SMX) interferes with DHF synthesis in
o strands prokaryotes, co-trimoxazole = TMP + SMX), inhibited by pyrimethamine in protozoa.
 antiparallel Result = ↓ dTMP
 right handed double helix Deficiency: orotic aciduria: inability to convert orotic acid to UMP, defect in uridine
o Chargaff's rules monophosphate (UMP) synthase; AR. Presentation: ↑ orotic acid crystals in urine,
 A % = T/U % megaloblastic anemia (does not improve with administration of vitamin B12 or folic acid:
 G%=C%
not enough thymidine to sustain normal erythropoiesis), failure to thrive, no DNA gyrase (topoisomerase II) breaks the DNA to prevent coiling.
hyperammonemia (distinguishes between ornithine transcarbamylase (OTC) deficiency RNA primer removed by RNAase H in eukaryotes and filled by a DNA polymerase by DNA
with high [orotic acid] in urine with hyperammonemia). Treatment: oral uridine polymerase I in prokaryotes and can fill simultaneously.
administration, bypasses defect in de novo pyrimidine pathway DNA ligase seals the nick between fragments.

Purine: pathway diagram, insufficient capacity in most cells, important enzymes: PRPP Differences between prokaryotes eukaryotes: prokaryotes (single origin of replication),
amidotransferase: rate-limiting step, inhibited by AMP, GMP, IMP, indirectly inhibited by eukaryotes (multiple origins of replication)
allopurinol, 6-mercaptopurine. Note: base of inosine = hypoxanthine Clinical importance: antibiotics (quinolones, fluoroquinolones block bacterial
Nucleotide catabolism/salvage: topoisomerase; used to treat aerobic gram negatives in UTIs and gonorrhea; e.g. drugs
Purine: salvage pathway (see figure), clinical importance: ending in –floxacin), cancer chemotherapy (etoposide, teniposide block eukaryotic
Adenosine deaminase (ADA) deficiency: defective purine salvage, results in excess ATP topoisomerase)
and dATP, prevents DNA synthesis. ATP and dATP feedback negatively on ribonucleotide
reductase in the synthesis of purines and pyrimidines for DNA replication, ↓ lymphocyte Reverse transcription: Function: convert RNA to DNA, performed by reverse
count (major cause of SCID (severe combined immunodeficiency disease: lack of both T transcriptase, has high error rate and cannot proofread, three activities: RNA-dependent
and B cells); AR DNA polymerase, RNase, DNA-dependent DNA polymerase
Lesch-Nyhan disease: defective purine salvage; lacks hypoxanthine guanine Clinical relevance: present in retroviruses e.g. HIV, some HIV drugs inhibit reverse
phosphoribosyl pyrophosphate transferase (HGPRT) enzyme; XR, presentation: severe transcriptase: AZT (enters cells and modified by addition of ATP, functions as a
CNS symptoms (choreoathetosis, mental retardation), self-mutilation, hyperuricemia nucleotide analog, terminates replication). Also present in telomerase; used in RT-PCR.
(degradation of all purines since it cannot salvage)
Gout: pathophysiology: high urate levels due to: ↑ in cell breakdown (e.g. treatment of DNA repair:
large tumor masses with radiation or chemo), ↓ in renal excretion (most common cause). Damage tolerance:
Results in precipitation of monosodium urate crystals in joints, negative birefringence  Function
(yellow when parallel to slow ray; needle shaped). Presentation: recurrent acute o can allow DNA replication to continue despite presence of DNA
arthritis: pain in big toe first (podagra), chronic: tophi present (granulomatous damage (e.g. thymidine dimer)
deposition (multinucleated giant cells) of crystals in soft tissue; ↑ frequency in men >30  Process
y/o. Treatment: acute → colchicine or indomethacin, chronic due to ↓ in renal excretion o DNA polymerase stalls at dimer
→ probenecid, chronic due to ↑ in cell breakdown → allopurinol o sliding clamp releases regular DNA polymerase and binds the one of
two translesion polymerases
Pyrimidine: salvage (may be salvaged by pyrimidine salvage enzymes), degradation  error free
(completely broken down to ammonia)  recognizes that the dimer is normally a thymidine
Other causes of hyperuricemia: and the polymerase adds an adenosine opposite
↑ EtOH intake: can precipitate an acute gout attack and continues replication
↑ nucleic acid in diet: red meats, organ meats  error prone
Phosphate "trapping" diseases:
 polymerase adds any base opposite the lesion and
▪ e.g. glucose-6-phosphate deficiency (G6PD), galactose uridyltransferase
continues replication
deficiency
mismatch repair:
▪ caused by an inability to dephosphorylate common metabolites and therefore
leads to trapping of phosphate by these metabolites  Process
▪ lack of phosphate prevents synthesis of ATP, GTP, plus other nucleotide o repairs G/T or A/C pairing
phosphates. ADP, AMP and other hypophosphorylated bases are salvaged  sometimes misincorporated due to tautomerization of the
producing uric acid nucleotide
o involves MutS, MutH, MutL enzymes
DNA replication: o strand specific
Function: replicate cell genome in a manner that is highly accurate  recognizes which is the new strand because it is
Process: DNA melted to expose single strand to expose origin of replication, single unmethylated and the old strand is methylated
stranded binding proteins (SSBs) bind and stabilize melted DNA, RNA primer added in 5'  Deficiency
→ 3' direction by primase, DNA polymerase adds adds nucleotides in a 5' → 3': o hereditary nonpolyposis colorectal cancer
DNA polymerase III in prokaryotes, DNA polymerase α and δ in eukaryotes, can edit  aka Lynch syndrome
mistakes with a 3' → 5' exonuclease activity, adds continuously on the leading strand,  cause
adds discontinuous Okazaki fragments on the lagging strand because it must synthesize  hereditary absence of one copy of
in a 5' → 3' direction. enzyme hMLH1 or hMSH2
 second copy lost due to somatic mutation due to the use of the opposite strand as a template)
 known as the two-hit model Deficiency: Bloom syndrome: cause: lack of BLM helicase enzyme; presentation: short
 common to many DNA repair stature, rash from sun exposure, café-au-lait spot, leukemias, lymphomas, carcinomas
deficiencies BRCA-1 involved in: breast, prostate, ovarian cancer
 presentation BRCA-2 involved in: breast cancer
 microsatellite instability
 di-, tri-, tetranucleotide repeats that can Non-homologous end joining:
be amplified Function: repair double-strand breaks (these breaks may be caused by ionizing radiation
 constant in number in normal or oxidative free radicals; mechanism of cancer radiation therapy), occurs when a sister
cells chromatid is not available to use as a template (prior to S phase of cell cycle)
 diagnostic in Lynch syndrome Process: break recognized by MRN complex, additional enzymes (Artemis, XLF, Pol μ) cut
 ↑↑ risk of colorectal cancer ends so they can bind, DNA ligase IV joins ends together
 NOT preceded by benign polyps Deficiency: severe combined immunodeficiency disease (SCID) (one of many causes)

Base excision repair: Telomeres:


Function: specific endonucleases (glycosylasse) remove bases that have been modified  Structure
by several common mechanisms of damage e.g. deaminated cytosines (C → U) removed o regions of DNA at the ends of chromosomes that are non-coding and repetitive
by uracil glycosylase; can take place anytime during the cell cycle but occurs primarily in o added by telomerase enzyme
G1  Function
Process: glycosylase specific for the damaged nucleotide removed damaged base by o protect ends of DNA from degradation
breaking glycosidic bond, damaged base removed (sugar remains but base removed:  Clinical relevance
creates an apurinic/apyrimidinic (AP) site), gap filled by DNA polymerase, ligation of o each round of replication shortens the telomere because DNA polymerase requires
strand nick by DNA ligase III an RNA template and lagging strand cannot replicate all the way to the end of the
chromosome
Nucleotide excision repair: o telomere shortening is implicated in cell senescence
Function: removes thymidine dimers caused by UV-B light, removes damaged bases o cancer cell often have ↑ telomerase activity
caused by chemicals  makes unlimited replication potential possible
Process: maintenance repair: RNA:
• XPC recognises DNA lesion and recruits XPA  rRNA
• XPB-G binds DNA and removes a chunk spanning the damaged segment o ribosomal
• DNA polymerase fills the gap o rRNA + protein = ribosome
• DNA ligase seals the nick o most abundant of all RNA types
transcription-coupled repair o clinical importance
• RNA polymerase stalls at DNA lesion  shiga- and verotoxins bind to 28S rRNA
• CSB and XPG recognize stalled RNA polymerase  inhibit eukaryotic protein synthesis
• CSA joins complex and removes damaged site and allows transcription to continue  tRNA
Deficiency: xeroderma pigmentosum (XP): cause: lack any enzyme XPA – XPG. o transfer
Presentation: cannot repair UV damage: sunlight sensitivity, ↑↑↑ prevalence of skin o covalently attached to amino acids
cancer, corneal ulcers. Diagnosis: measurement of repair mechanisms in white blood o provides link between genetic code and a particular amino acid
cells. Treatment: avoidance of sunlight
 mRNA
Cockayne syndrome: cause: lack of CSA or CSB; AR; presentation: growth failure,
o messenger
photosensitivity, nervous system abnormalities; can affect any organ system
o carries DNA message from chromosome to translation by ribosome
 hnRNA or pre-mRNA
Homologous recombination: o heteronuclear
Function: repair double-strand breaks, requires a sister chromatid to use as a template o present only in eukaryotes
(therefore must occur after S phase of cell cycle) o mRNA transcript prior to splicing
Process: double-strand break recognized by MRN complex, BRCA and BLM enzymes  contains introns and exons
involved in end processing, Holliday junctions are formed (cross-shaped structures that  snRNA
mediate strand rejoining), junctions are resolved (may result in loss of heterozygosity, o small nuclear
o present only in eukaryotes  α-amanitin
o functions in splicing of mRNA in the nucleus  poison in death cap mushrooms
 Ribozymes  actinomycin D
o present in eukaryotes and prokaryotes  chemotherapeutic
o have intrinsic enzymatic activity Gene organization:
 miRNA Enhancer: function: allow signal transduced to ↑ transcription, act by binding and
o micro bending DNA such that they can act on the promoter directly. Location: may be very far
o ↓ translation by binding to 3'-UTR of target mRNA from gene locus, may be within gene locus, orientation does not matter. Examples:
 double-stranded RNA signals for degradation glucocorticoid response elements (GREs) bind glucocorticoid, estrogen response
o uses Dicer and RISC enzymes in processing elements (EREs) bind estrogen, cAMP response element (CREs) bind cAMP
o often coded for in intron segments Silencers: act in a mechanism similar to enhancers but ↓ transcription
o RNAi is a synthetic lab technique that takes advantage of miRNA
pathway Upstream promoter elements: function: allow transcription factor induced transcription
RAN polymerase: ↑. Location: close to gene locus; examples: CCAAT box bind NF-1, TATA box
Prokaryotes:
 1 type of RNA polymerase (RNAp) 5'-UTR: untranslated region upstream of start codon
 Structure
Kozak consensus sequence: GCCGCCRCCAUGG, where R is A or G three bases upstream
o α2ββ' core subunits
from AUG start codon; plays a major role in initiation of translation, a mutation in the
 Initiation Kozak consensus sequence may cause β-thalassemia intermedia)
o σ (sigma) = initiation factor
o binds RNAp to DNA Start codon: codes for methionine (AUG), beginning of translated (coding) region
 Termination Coding region: between start codon and stop codon, following transcription contains
o ρ (rho) = termination factor introns and exons (eukaryotes)
 Clinical importance
o inhibited by Stop codon: UGA, UAG, UAA; terminates translation; 3'-UTR: untranslated region
 rifampin downstream of stop codon
 antibiotic
 actinomycin D Transcription: Process:
 antibiotic Initiation: RNAp binds the template DNA strand at the promoter site with the help of
eukaryotes: transcription factors
 3 types of RNA polymerase Elongation: RNAp synthesizes the mRNA strand in 5' → 3' direction (reads template
o RNAp 1 strand in 3' → 5' direction; eukaryotic RNAps transcribe introns as well as exons), RNAp
 synthesizes rRNA in nucleolus inhibited by alpha-amanitin (ingestion of the amanita phalloides mushroom)
 remember: rRNA is most abundant RNA; so it is #1 Termination:
o RNAp 2 prokaryotes: two mechanisms:
 synthesizes mRNA, snRNA in nucleus rho-dependent: rho protein binds the mRNA; causes RNAp to fall off
o RNAp 3 rho-independent: mRNA has a GC-rich segment that naturally forms a hairpin; causes
 synthesizes tRNA in nucleus RNAp to fall off
 remember: tee = three eukaryotes: transcription continues past the gene and eventually falls off
 makes eukaryotic 5s rRNA
Mono/polycistronic: monocistronic = mRNA codes for 1 protein (eukaryotes only have
 Structure monocistronic mRNA), polycistronic = mRNA codes for >1 protein (e.g. bacterial lac
o similar in structure to prokaryotes operon)
 Initiation Transcription regulation:
o TFIID = initiation factor Function: can turn transcription on/off, can ↑ or ↓ rate of transcription, can act in cis or
o binds RNAp to DNA trans:
 Termination cis = regulation near gene locus: DNA binding sequence; a mutated cis regulatory
o translation continues far past gene and RNAp eventually falls off element can result in gene that is constitutively on or off
 Clinical importance trans = regulation from gene locus: transcription factor protein; because it acts at a
o inhibited by distance a good allelic copy can compensate for a mutated copy
- CCU - CAU - 3' mRNA codes for (N-term) - Phe - Pro - His - (C-term) peptide
Mechanisms of control: modify RNAp binding stability, transcription factors:
Function: modify basal transcription levels, two types: Amino acids:
General: must bind to DNA and RNAp to begin baseline transcription of most every gene Structure: carbon attached to carboxyl group + amino group + "R" group. "R" group gives
e.g. TFIID binds TATA box and RNAp II each of the 20 amino acids its unique properties: only L-form amino acids (AA) are found
Specific: acts through enhancers and silencers; can regulate specific gene responses. in proteins.
Structure: DNA binding domain (can be zinc fingers, helix-turn-helix, helix-loop-helix, or
leucine zippers), regulatory element binding domain (e.g. binds hormone, ion, other Properties:
transcription factors, etc). hydrophobicity: allow for protein folding such that hydrophilic AAs face externally and
Modify RNAp accessibility to DNA: hydrophobic AAs face internally, hydrophobic AAs have aliphatic or aromatic R-groups,
histone modifiers: histone acetylases (HATs) open DNA and ↑ transcription, histone hydrophilic AAs have oxygen or nitrogens in R-group.
deacetylase (HDACs) close DNA and ↓ transcription pH: acidic AAs are negatively charged at body pH (Asp, Glu), basic AAs are positively
Imprinting: methylation effectively shuts a gene off; often irreversible, some genes charged at body pH (Arg, His, Lys)
methylate a gene locus on paternal or maternal chromosome, allows only one allele to be essential/non-essential: essential AAs must be part of the diet. Remember: PVT TIM
active e.g. Prader-Willi/Angelman syndrome HALL (Phe/Val/Thr/Trp/Ile/Met/His/Arg/Lys/Leu)
inactivation of a chromosome: condensation of # of X chromosomes - 1 to form Barr catabolic product:
bodies e.g. Turner's Syndrome (XO) the patient would have no Barr bodies as they only glucogenic AAs can become glucose (Met, Val, Arg, His)
have 1 X chromosome ketogenic AAs can become ketone bodies (Leu, Lys)
increase number of gene copies: more sites for RNAp to bind; common in oncogenes Some AAs can become both ketone bodies or glucose (Ile, Phe, Thr, Trp)

Embryonic gene regulation: AA derivatives:


sonic hedgehog (SHH) gene: mutations causes holoprosencephaly (HPE); failure of Tyrosine: thyroid hormones (T3/T4), catecholamines, melanin:
midline brain to separate into right and left Deficiency: albinism; cause: defective tyrosinase (inability to synthesize melanin from
homeobox (HOX) genes: control proper timing of gene activation tyrosine), defective tyrosine transporters (↓ amounts of tyrosine and thus melanin), lack
paired box (PAX) genes: mutations cause Klein-Waardenburg syndrome; presentation: of migration of neural crest cells forming melanocytes; presentation: lack of melanin ↑
neural crest abnormalities, deafness, variation in pigmentation (forelock of white hair, risk of skin cancer. Genetics: AR, locus heterogeneity can result in variable inheritance:
patches of different colored skin), dystopia canthorum (broad nasal root) XR for ocular albinism
Phenylalanine: catecholamines
Examples of gene regulation: Tryptophan: NAD, NADP (niacin), serotonin
peroxisome proliferator-activated receptors (PPARs): controls fat metabolism: turned on Histidine: histamine
by endogenous ligands (fatty acids, prostaglandins), also turned on by exogenous ligands Arginine: nitric oxide (NO)
(fibrates, thiazolidinediones); bind PPRE region in DNA Glutamate: GABA
Clinical importance: fibrates given to hyperlipidemic patients to ↑ transcription of Glycine: protoprophyrin → heme
lipoprotein lipase also used in treatment for Zellweger syndrome.
Amino acid absorption:
RNA processing: Only in eukaryotes; pre-processed transcript = hnRNA Nitrogen balance:
Process: Positive: amino acid (AA) intake > excretion; examples: growth, pregnancy
Capping: 5' end (7-methylguanosine cap, occurs before transcription has finished, Negative: AA excretion > intake; examples: kwashiorkor, starvation, infection
functions to help in ribosomal binding, also protects against degradation), 3' end
(polyadenylated (poly-A) tail, added by poly-A-polymerase, functions to protect from AA absorption in intestine:
degradation, some viruses can steal host cell caps so that the viral mRNA gets translated) Proteins degraded by trypsin and pepsinenteropeptidase cleaves trypsinogen (inactive)
Splicing: process of intron removal, mediated by the spliceosome (snRNPs) (lariat is to trypsin (active)
formed, adjacent exons are attached). Absorbed as AA's across the gut lumen with specific transporter for similar AA's; e.g.,
Alternative splicing: one hnRNA can be spliced differently to produce different protein transporter for basic AA's, transporter for large neutral AA's.
products Deficiencies: lack of absorption with pancreatitis due to ↓ in degradative enzyme
Location: occurs in nucleus after transcription; only processed RNA is transported out of production. Hartnup disease: defect in large neutral AA transporter in the intestine and
the nucleus renal proximal tubule cells, result is a tryptophan deficiency, presents similar to pellagra

AA absorption in kidney: AA's that are filtered from the glomerulus can be actively
Protein synthesis: function: converts mRNA nucleotidemessage into a polypeptide. reabsorbed in the proximal convoluted tubules with similar transporters as the gut;
Location: mRNA must move from nucleus to cytoplasm for translation. Example: 5' - UUC deficiencies: cystinuria: genetic defect in transporter for cysteine, ornithine, lysine,
arginine; AR; presentation: cystine staghorn calculi: note: cystine = 2x cysteine attached aspartate - NH3 = oxaloacetate
by a disulfide bridge, cystine kidney stones result from high concentrations in urine. glutamate - NH3 = α-ketoglutarate
Treatment: alkalinization of the urine with acetazolamide.
Defects in specific amino acid catabolism: all are part of newborn screening program:
Amino acid catabolism: Three possible fates: enter citric acid cycle, form ketone bodies, phenylketonuria (PKU): inability to break down phenylalanine, deficient in
substrates for gluconeogenesis phenylalanine hydroxylase, ↓ tetrahydrobiopterin cofactor. Presentation: ↑
phenylalanine, ↓ tyrosine, requires tyrosine supplementation, mental retardation,
Urea cycle: function: degrade excess amino acids and safely remove nitrogen, (surplus microcephaly, musty/mousy odor to sweat and urine, restriction of phenylalanine in the
amino acids cannot be stored), produce urea. diet though cannot eliminate as it essential for protein synthesis, very strict adherence to
Pathway: aspartate and carbamoyl phosphate provide nitrogens (carbamoyl phosphate diet during pregnancy for a mother with PKU, avoid aspartame
synthesized from NH4+ + HCO3- + 2 ATP via carbamoyl phosphate synthetase I, rate
determining step of pathway, requires N-acetylglutamate which regulates the cycle, only maple syrup urine disease: inability to breakdown branched-chain amino acids (Val, Leu,
produced when excess amino acids are present), nitrogen added from systemic pool via Ile), deficient in branched-chain ketoacid dehydrogenase. Presentation: infantile onset
alanine cycle, one turn of the cycle: aspartate + NH3 + CO2 + 3 ATP → urea (containing (normal for first week, progressive onset of symptoms), lethargy, weight loss,
2N)+ fumarate + 2 ADP + Pi + AMP + PPi + 3 H20, connected to citric acid cycle via hyper/hypotonia, mental retardation, urine smells of maple syrup, death if dietary intake
aspartate-argininosuccinate shunt (fumarate of urea cycle → malate of citric acid cycle, of Val, Leu, Ile is not restricted
oxaloacetate of citric acid cycle → aspartate of urea cycle).
Location: Cellularly: formation of carbamoyl phosphate occurs in the mitochondrial alkaptonuria: inability to breakdown homogentisic acid (breakdown product of tyrosine
matrix; addition of aspartate and removal of fumarate and urea occurs in the cytoplasm. and phenylalanine), deficient in homogentisate oxidase. Presentation: arthritis
Systemic: liver and kidney (accumulates over years in the cartilage (ochronosis), onset prior to third decade, urine
Deficiencies: common presentation: hyperammonemia + ↑ [glutamine]blood + ↓ blood urea that darkens upon sitting in air, dark coloration of the sclera
nitrogen (BUN), onset shortly after birth (< 1-3 day), hyperammonemia, intoxication
presents with cerebral edema, vomiting, hyperventilation, lethargy, blurring vision; α- Hartnup's disease: deficiency of neutral amino acid transporter, leads to ↓ tryptophan
ketoglutarate consumed, stops TCA cycle absorption. Presentation: pellagra, result of niacin deficiency (niacin produced from
tryptophan)
carbamoyl phosphate synthase I creates carbamoyl phosphate: AR inheritance pattern;
orotic aciduria absent Homocystinuria: inability to breakdown homocystinuria (methionine degradation
pathway). Causes: cystathionine synthase deficiency, ↓ affinity of cystathionine synthase
ornithine transcarbamoylase forms citrulline from carbamoyl phosphate; XR inheritance for pyridoxal phosphate (B6), homocysteine methyltransferase deficiency, deficiency in
pattern, most common urea cycle disorder. Orotic aciduria because excess carbamoyl folate, B6 or B12 in the diet can produce a less severe form of homocystinuria.
phosphate is shunted into the UMP synthetic pathway in which orotic acid is an Presentation: vessel damage (DVT, atherosclerosis, MI before 2nd decade of life), similar
intermediate. to Marfan's: mental retardation, lens dislocations downward as opposed to upward in
Marfan syndrome, tall with long extremities, ↑ homocysteine in the urine.
Treatment: low protein diet, benzoate or phenylbutyrate (chelate nitrogen by becoming Treatment varies by cause: cystathionine synthase deficiency (↓ intake of Met, ↑ intake of
aminated) Cys, B12 and folate), ↓ affinity of cystathionine synthase for pyridoxal phosphate (↑ intake
of B6)
Ammonia transport: function: safely move nitrogenous wastes from tissues to kidney
and intestine in the form of glutamine. Pathway: ammonia loaded via glutamine propionyl-CoA carboxylase/methylmalonyl-CoA deficiency: inability to handle Val, Met,
synthetase (NH3 + glutamate → glutamine; occurs in nearly all tissues), ammonia Ile, Thr; part of propionic acid pathway. Presentation: ketoacidosis, propionyl-CoA
unloaded via glutaminase (glutamine → NH3 + glutamate, specific to kidneys and carboxylase deficiency has ↑ propionic acid, methyl citrate, hydroxypropionic acid,
intestine (and low concentration in liver), induced by acidosis). methylmalonyl-CoA mutase deficiency has ↑ methylmalonic acid. Treat by restricting Val,
Met, Ile, Thr in the diet
Glucose-alanine cycle: function: transport pyruvate from muscle to liver for
gluconeogenesis. Pathway: involves reversible aminotransferase reactions tRNA: Structure: cloverleaf structure, 3' aminoacyl end = CCA, bound covalently to amino
acid (AA)
alanine aminotransferase (ALT): glutamate + pyruvate → α-ketoglutarate + alanine in Charging: purpose: adds correct AA to correct tRNA: each AA has a specific tRNA, correct
muscle; α-ketoglutarate + alanine → glutamate + pyruvate in liver, requires vitamin B6 AA is only checked when attachment occurs by aminoacyl-tRNA synthetase to the tRNA,
thus, if a AA is mischarged, it has no other correction mechanism and will insert for the
aspartate aminotransferase (AST): glutamate + oxaloacetate → α-ketoglutarate + AA specified by the tRNA anticodon. Charged tRNA contains the needed energy for the
aspartate in liver; relationship between amino acids and α-keto acids: formation of the peptide bond. Process: tRNA + ATP + aa → aa-tRNA + AMP + PPi,
alanine - NH3 = pyruvate catalyzed by aminoacyl-tRNA synthetase, inhibited by tetracyclines
2 GTP to GDP for elongation (2)
Wobble: first 2 nucleotide positions provide specificity for an individual amino acid e.g.,
mRNA triplets CUU, CUC, CUA, CUG all code for Leu. Genetic code is degenerate, all amino Pharmacological inhibition of prokaryotic protein synthesis:
acids except for one have more than one codon (explains silent mutations: mutation from  Bind 30S
CUU to CUC would not alter the protein) o aminoglycosides
 inhibit formation of the initiation complex
Protein synthesis:  promotes misreading of mRNA
Ribosome: eukaryotes: small subunit (40S) synthesized in the nucleus, large subunit  Bind 50S
(60S) synthesized in the nucleolus, total = 80S, S = sedimentation; prokaryotes: small o chloramphenicol
subunit (30S) + large subunit (50S) → 70S  inhibits peptidyl transferase
o macrolides
Catalytic sites :
 inhibits translocation
A (acceptor) site: accepts charged aa-tRNA
o clindamycin
P (peptidyl) site: holds the growing peptide chain and transfers to the aa-tRNA in the A
 inhibits translocation
site
E (exit) site: allows tRNA to exit the ribosome after releasing its amino acid to the aa-
Protein structure:
peptide chain
Peptide bond: formation: carboxyl group on one amino acid + amino group on another
amino acid, results in a loss of water, bond can be broken (hydrolyzed) with addition of
Forms: polysomes: single mRNA being translated simultaneously by several ribosomes,
water across the bond
can be: free (proteins for nucleus or mitochondria), membrane bound (found on rough
ER, proteins for secretion or membrane insertion)
Orders of protein shape:
Primary: amino acid sequence determined by covalent peptide bonds
Steps:
Secondary: stable folding of individual protein domains (a protein may have
Initiation:
combinations of different secondary structures), common forms (α-helix, β-pleated
initiation complex generated from:
sheet), determined by amino acid-amino acid interactions via hydrogen bonds
small ribosomal subunit
Tertiary: shape of protein as a whole which imparts functionality to a protein (shape
mRNA: 5'-cap (eukaryotes), Shine-Dalgarno sequence (prokaryotes) may be disrupted (denatured) with changes in solution), common forms (globular,
initiation aa-tRNA: met-tRNA (eukaryotes), fmet-tRNA (prokaryotes) fibular), determined by h-bonding, hydrophobicity, disulfide bridges, ionic bonds
initiation factors (IFs): help in assembly, released when mRNA joins the ribosomal Quaternary: combination of tertiary sub-units, examples: α + β subunits in hemoglobin
subunit
Once assembled, recruits the large ribosomal subunit: initiation aa-tRNA occupies the P- Proetein folding and degradation:
site (all other aa-tRNAs bind the A-site), the A-site is empty Folding:
 Overview
Elongation: aminoacyl-tRNA binds to A site (consumes 1 GTP), peptidyl-bond formation
o required for a protein to achieve a proper tertiary protein structure
(peptidyl transferase reaction catalyzed by ribosomal rRNA ("ribozyme") , transfers
o involves heat shock proteins (Hsp)
growing polypeptide to amino acid in the A site)
 essential for normal protein folding
Translocation: ribosome moves 3 nucleotides at a time in a 5' to 3' direction: moves  some function as chaperones and some function as
peptidyl RNA from A site to P site, moves empty tRNA from P site to E site for exit, A site chaperonins
now empty and ready to accept next aa-tRNA, catalyzed by elongation factor-2 (eEF-2) o the more mutated a protein, the more help it needs from chaperones
(inactivated via ADP-ribosylation by bacterial toxins (Pseudomonas and Diphtheria)), o if a protein is not folding properly, a chaperone may send it directly
consumes 1 GTP. Cycle is repeated for each amino acid addition to the polypeptide for degradation
Wobble phenomenon: The anticodon of certain tRNA molecules can bind to multiple o clinical relevance
codons. The first two nucleotides of the codon adequately specify which tRNA (and  cystic fibrosis
amino acid) binds the codon, thereby permitting multiple nucleotides in the third  pathogenesis
position to bind the same tRNA.  3 nucleotide deletion on chromosome 7
 ΔF508 mutation in chloride channel
Termination: stop codon reached in mRNA, release factor binds mRNA, peptide bond is (CFTR) ↓ stability of the protein and ↑
hydrolyzed and completed protein is released, ribosomal complex dissociates folding time

Overall energy requirement: 4 high energy bonds: ATP to AMP for tRNA charging (2)
 instead of insertion into the plasma  Defects in destruction of misfolded proteins
membrane the protein is degraded in the o inability to send degraded proteins to proteasome results in
Golgi apparatus accumulation in ER
 ↓ chloride conductance results in ↓ o examples
Na+ and Cl reabsorption in sweat glands  α1-antitrypsin (AAT) deficiency
 presentation  normally synthesized by hepatocytes and
 ↓ water content of mucus which results in exocytosed into circulation
a thick mucus that cannot be cleared  inhibit proteases
 respiratory infections  in AAT deficiency misfolded α1-antitrypsin
 nasal polyps accumulates in ER and damages hepatocytes
 malabsorption  PAS+ granules
 meconium ileus  many genetic variations
 biliary cirrhosis  MC are Z and S variants due to point
 Chaperones mutations
o types  co-dominant allelic expression
 Hsp70  presentation
 associates with directly with the ribosome  micronodular cirrhosis
 hides hydrophobic regions of protein to allow for  fibrosis
proper folding  test with PCR
 ATP hydrolysis required
 essential post-translational modification:
 Hsp90  Covalent alterations
 used for fewer proteins than Hsp70 o phosphorylation
 ATP hydrolysis required  kinase additions of phosphate groups
 essential o glycosylation
 role in folding mutant proteins in cancer  addition of oligosaccharide
 Chaperonins  signals for translocation into ER/Golgi
o group 1 o hydroxylation
 Hsp60  allows for ↑ in hydrogen bonding
 ring shaped  needed for collagen synthesis
 ATP hydrolysis required o γ-carboxylation
 called GroEL/GroES in prokaryotes  addition of carboxyl group
 peptide chain enters the cage and it is capped  allows for Ca2+ binding site
 once folded the cap is removed and the protein is  needed for several clotting factors
released o prenylation
o group 2  addition of farnesyl or geranylgeranyl lipid groups
 TRiC/CCT  signals for membrane insertion
 composed of 8 Hsp60s  Proteolysis
 similar function to GroEL/GroES o cleavage of N- or C- terminal propeptides
 required for folding of actin and tubulin  peptides beginning in "pro-"
o converts zymogens to mature enzymes
degradation:  examples
 Ubiquitination  enzymes ending in "-ogen"
o cell's mechanism to mark a protein for destruction
o mechanism types of DNA mutation:
 several copies of ubiquitin added to a misfolded/unneeded silence:
protein  Exchange of one base for another results in same amino acid
 polyubiquitinated protein enters the proteasome o often an alteration in 3rd position of codon
 protein hydrolyzed into peptide fragments  tRNA wobble
 No change in protein function Deficiencies in heme synthesis: hemin (occurs when Fe3+ is incorporated instead of Fe2+),
porphyria (causes symptoms due toxic accumulation of pathway intermediates:
Missense: Exchange of one base for another results in changed amino acid: aminolevulinic acid (ALA) causes neurological symptoms, protoporphyrins cause
transversion → exchanges a purine to a pyrimidine or a pyrimidine to a purine e.g., C → A photosensitivity: conjugated structure what absorbs light energy and forms free
or G → T radicals), symptoms worsened by: sunlight, P450 inducing drugs (stimulate the heme
transition → exchanges a purine for another purine or a pyrimidine to another synthesis pathway to ↑ production, ex: barbiturates, alcohol). Treatment: limit exposure
pyrimidine e.g., A → G or C → T to sun and P450 inducing substances, hemin (inhibits new heme production). Types:
Variable change in protein function: if the new amino acid is similar to old (leu → ile) the porphyria cutanea tarda: deficiency in uroporphyrinogen decarboxylase; AD, late onset
protein will most likely function the same, if the new amino acid is different (glu → val) (4th or 5th decade), symptoms often noticed with alcohol consumption. Presentation:
the protein folding/stability will likely be affected e.g., sickle-cell anemia (glu → val photosensitivity, hyperpigmentation (body's attempt to protect the skin), dark
mutation in β-globin gene) red/brown colored urine
acute intermittent porphyria: deficiency in porphobilinogen deaminase; AD, late onset.
Nonsense: Presentation: NO photosensitivity, episodic psychological symptoms (paranoia, anxiety,
 Exchange of one base for another results in a stop codon depression), vague abdominal pain (patients can present with a history of laparoscopies,
dark red/brown colored urine, ALA and porphobilinogen (PBG) present in urine during
 Loss of function mutation as peptide is truncated
symptoms)
Frameshift:
• Deletion or addition of 1 or 2 bases resulting in misreading of all nucleotides poisoning:
downstream lead: induced deficiency in ALA dehydratase and ferrochelatase, both enzymes are Zn2+
• Loss of function mutation as peptide is completely different dependent metalloenzymes. Pb2+ replaces the Zn2+ at the active site. Presentation: ↓ in IQ
(microcytic anemia with coarse basophilic stippling), abdominal pain (↑ in ALA without ↑
in PBG, differentiates from porphyrias), lead lines in bone and teeth xrays,
Large segment deletion: Unequal crossover at meiosis results in loss of large segment of
nephrotoxicity (deposition in nuclei of proximal renal tubular cells)
DNA
 Loss of function mutation hexachlorobenzene: induced deficiency in uroporphyrinogen decarboxylase;
 e.g., α-thalassemia (deletion of α-globin gene) presentation: hypertrichosis (↑ body hair coverage), found in (now banned in USA)
pesticides.
change at splice site: iron deficiency (iron incorporated in the final step), result is microcytic hypochromic
 Alteration in base sequence at mRNA splicing site results in altered splicing: anemia
o can remove parts of exon vitamin B6 deficiency (rate limiting enzyme (ALA synthase) requires), most commonly
o can leave parts of intron due to isoniazid therapy
 Variable effect on protein function as number of spliced amino acids varies
 e.g., β-thalassemia degradation of heme:
 Function
triple repeat expansion: o rid body of hemoglobin removed from degraded RBCs
 Expansion of short nucleotide sequence results in longer polypeptide  Location of synthesis
o can be in coding or noncoding region o spleen
 Addition of amino acids affects protein structure/folding and affects function  site of RBC destruction
 Disease display anticipation o liver
o earlier disease onset in successive generations  site of bilirubin conjugation
 e.g., myotonic dystrophy, Huntington's disease, and Fragile X o intestine
 conversion by normal gut flora
In-frame mutations
bilirubin:
Heme metabolism:  Properties
Heme synthesis: function: hemoglobin, cytochrome b4, P450 o insoluble
Location of synthesis: involves both the mitochondria and the cytosol, occurs in nearly  must travel in blood bound to albumin
every cell, occurs in RBC progenitor cells (CANNOT occur in RBCs because they lack o conjugation
mitochondria)  direct (conjugated)
Pathway: ALA synthase is rate limiting enzyme (see above)  glucuronate group added
 soluble Polygenic inheritance: multiple genes are responsible for inheritance of a disease e.g.)
 indirect (unconjugated) androgenic alopecia
 glucuronate group not yet added
 insoluble Heritability: can measure the relative effect of genetic vs. environmental factors on a
 Modified forms phenotype, calculated by phenotypic relationship between dizygotic (DZ) and
o urobilinogen monozygotic (MZ) twins, heritability = (CMZ-CDZ) / (1-CDZ), where C = concordance.
 gives urine yellow color Prevalence of disease in both twins entirely environmental disease should have CMZ =
o stercobilin CDZ, entirely genetic disease should have CMZ=1.0 and CDZ=0.5; siblings share 50% of their
 gives feces brown color genes.
 with a blocked bile duct no stercobilin in feces and
Mode of inheritance:
it is clay colored
AD: Genders affected: male and female at equal frequency; does not skip generations (if
genetics:
two parents without the AD disease have child with an AD disease): Possibility is
reduced penetrance (have mutant gene but phenotypically normal), de novo germline
Codominance: both allelic copies are expressed e.g.) blood groups (A, B, AB)
mutation
Variable expression: nature and degree of phenotype vary from 1 individual to another
An affected child must receive disease from an affected parent: a homozygote dominant
with the same mutation e.g.) 2 patients with neurofibromatosis may have varying
parent has a 100% of having an affected child, two heterotyzgote parents with the AD
disease severity
disease condition have a 75% chance of having a child with the disease phenotype.
Incomplete penetrance not all individuals with a mutant genotype have diseased
Pathology: defects in structural genes. Presentation timing: usually after puberty
phenotype, explanation for a dominant disease "skipping" a generation, penetrance can
Other notes: often pleiotropic (several organ systems affected by single genetic defect),
be calculated by ( # with symptoms) / (# with disease genotype), must be figured into
only one copy of the defective gene is required to express the disease phenotype.
recurrance calculations: if parents have a 50% chance of giving defective gene but the
Examples: von Willebrand disease (most common), Huntington's disease, osteogenesis
penetrance is 50%, 0.5 x 0.5 x100 = 25% of recurrence, observed in recessive and
imperfecta, achondroplasia, Marfan syndrome, neurofibromatosis type, acute
dominant diseases
intermittent porphyria
Pleiotropy: single mutation has diverse effects upon several organ systems e.g.) PKU
AR: genders effected: male and female; generations affected:1/4 of offspring affected
Anticipation: changes in disease presentation in succeeding generations, ↑ severity,
when both parents are carriers, usually 1 generation
earlier onset, caused by trinucleotide expansion, region of repeating triplets expands in
Pathology: defects in enzymes. Presentation timing:infancy to childhood
each generation e.g.) Huntington's disease, fragile X, myotonic dystrophy, Friedreich
Other notes: most often more severe than AD, must have 2 defective copies of the gene,
ataxia
chances greatly increased with consanguinity
Examples: cystic fibrosis - deficiency in the chloride channel CFTR, inborn errors of
Loss of heterozygosity: "two-hit model": individual inherits or develops a mutation in
metabolism (PKU, von Gierke's, Pompe's, glycogen storage diseases, sphingolipidoses
one copy of gene, disease occurs when the the complementary allele is lost e.g. tumor
(except Fabry's), and mucopolysaccharidoses (except Hunter's)), sickle cell anemia,
suppressor diseases (Li-Fraumeni, retinoblastoma)
thalassemias, albinism, ARPKD, hemochromatosis
Dominant negative mutation: mutant gene product antagonizes wild-type gene product,
X linked recessive: genders affected: males must receive defective gene from carrier
exerts a dominant effect e.g.) common in multimeric proteins where one mutant subunit
mother, carrier mother's sons have 50% of having disease, affected males give copy to all
can change function of entire enzyme
of their daughters
Generations affected: skips generations, male-to-male transmission not allowed, diseases
De novo mutation: genetic disease in an individual with no familial history, recurrence
passes through carrier daughters
risk for offspring of same parents is low
Pathology: defects in enzymatic genes, similar to AR diseases
Presentation timing: usually after puberty
Locus heterogeneity: different mutations can produce the same phenotype e.g.)
Other notes: only one defective copy necessary for disease in males because males are
marfanoid habitus caused by Marfan's syndrome, MEN 2B, homocystinuria
hemizygous for X chromosome, two defective copies necessary for disease in females
(can be affected with just one defective copy if normal X chromosome is inactivated to
Heteroplasmy: presence of both normal and mutated mitochondrial (mt)DNA in the
Barr body, called manifesting heterozygotes, phenotype usually milder than affected
same cell; results in variable expression in mitochondrial inherited disease
males)
Examples: hemophilia A and B, Menke's disease, Duchenne muscular dystrophy, Lesch-
Uniparental disomy: offspring receives both copies of a chromosome from 1 parent, no
Nyhan syndrome, Ornithine transcarbamoylase deficiency, red-green color blindness
copies from the other parent, causes disease if the chromosome is usually imprinted
X linked dominant: genders affected: male and female at equal frequency
 Generations affected o attachment of mitotic spindle fibers
o does not skip generations  allows chromosomes to be pulled to opposite poles during
 only possibility is reduced penetrance anaphase
o females of affected fathers are always affected o variations in position
 male-to-male transmission not seen  metacentric
o male or females of affected mothers can be affected  centromere in the middle
 Pathology  submetacentric
o defects in structural genes  centromere offset slightly towards one end
 Presentation timing  acrocentric
o usually after puberty  near complete displacement of centromere to one
 Examples end
o hypophosphatemic rickets  Nomenclature
o Fragile X syndrome o long = q
o Alport syndrome o short = p
Mitochondrial inheritance: genders affected: male and females at equal frequency  remember: p = petite
Generations affected: does not skip generations, only transmitted from affected female: o translocation = t
gives to all offspring, due to the fact that the sperm do not contribute mitochondria to the o deletion = del
zygote types of chromosomal alteration:
Pathology: defects in electron transport/oxidative phosphorylation process, presents as Nondisjunction: homologous chromatids do not separate properly during meiosis (stage
neuropathies/myopathies (neurons and muscle cells require high amounts of energy and of nondisjunction affects gamete production outcome, nondisjunction in meiosis I results
depend on mitochondria) in 2 gametes with x 2 and 2 gametes x 0, nondisjunction in meiosis II results in 2 normal
Presentation timing: usually after puberty gametes, 1 gamete x 2 and 1 gamete x 0; zygote receiving 3 copies = trisomy, zygote
Other notes: variable expression due to heteroplasmy, a small percentage of receiving 1 copy = monosomy), risk greatly ↑ with ↑ in maternal age, more common in
mitochondria within a cell are affected leading to variable severity oogenesis than spermatogenesis
Examples: myoclonic epilepsy with ragged red muscle fibers, Leber hereditary optic
neuropathy, MELAS (mitochondrial encephalomyopathy, lactic acidosis, and stroke-like Translocation: exchange genetic info between nonhomologous chromosomes by
episodes) breakage and repair. Balanced: where exchanged fragment is still functional on another
chromosome, unbalanced: where exchanged fragment cannot function properly;
Inheritance algorithm: Does offspring with disease have a parent with disease? (Y/N) common in cancers. Types:
o if YES Robertsonian: balanced, always involve two acrocentric chromosomes (13, 14, 15, 21,
 dominant (does not skip generations) 22), results in loss of short arm and fusion of two long arms of different chromosomes,
 is there male-to-male transmission of disease? (Y/N) no clinical presentation because short arms of acrocentrics contain no vital info (is a
 if YES translocation carrier, problems with gametogenesis and therefore reproduction
 autosomal dominant (miscarriage, aneuploidy): depends on how chromosomes segregate during homologous
 if NO pair separation).
 do daughters of affected male have disease? (Y/N)
Reciprocal: exchange of DNA between two non-homologous chromosomes, as long as no
 if YES
DNA is lost the phenotype is normal for that generation is a translocation carrier
 X-linked dominant
 if NO Inversion: type of rearrangement where part of chromosome is inverted in orientation;
 mitochondrial types: pericentric (inverted chromosomal segment includes centromere, remember:
o if NO pericentric involves centromere), paracentric (inverted chromosomal segment does not
 recessive (can skip generations) include centromere)
 predominantly males with disease? (Y/N)
 if YES Ring chromosomes: causes: product of two breakage sites on the chromosome and the
 X-linked recessive segment lost circularizes; ends of chromosomes join circularizing entire chromosome
 if NO (usually lost during gametogenesis → monosomy)
 autosomal recessive
chromosomal structure: Centromere: Isochromosome: replication of one arm of a chromosome with loss of the other, p-q → p-
o holds sister chromatids together p'; lethal for autosomes, can be observed on sex chromosomes
 Repeat expansion
Deletions: loss of chromosome segment; types: terminal (end of chromosome), o CGG repeat on FMR1 gene
interstitial (within the chromosome)  Presentation
o mental retardation
Chromosomal disease: o autism
Down syndrome: most common chromosomal disorder, most common cause of o long face large jaw
congenital mental retardation; causes: o large ears
trisomy 21: nondisjunction (most common cause), occurs during anaphase of meiosis I, o large testicles (macroorchidism)
Robertsonian translocation, mosaicism (least common cause); presentation: appearance: friedrich’s ataxia:
short stature, hypotonia, unique facial structure (epicanthic folds, macroglossia, flat
 Inheritance pattern
profile, depressed nasal bridge), simian crease in palm, ↑ risk for congenital heart disease
o autosomal recessive
(combined ASD and VSD), AML (< 3 y/o), ALL (> 3 y/o) , Alzheimer's disease (by 5th
decade, due to amyloid precursor protein (APP) gene on 21), Hirschsprung's disease,  Repeat expansion
duodenal atresia, congenital heart anamolies (atrioventricular canal is most common,
o GAA repeat on chromosome 9
endocardial cushion defects also very common). Screening: + quad screen (↓ α-  results in
fetoprotein, ↓ estriol, ↑ inhibin A, ↑ β-hCG), remember: high (hCG, inhibin); deficit (estriol,  defect in frataxin (an iron binding protein) that
fetoprotein) + ultrasound shows high amount of fluid behind the neck (↑ nuchal leads to impaired mitochondrial function
translucency), can confirm diagnosis with amniocentesis or chorionic villus sampling  degeneration of various spinal cord tracts
 Presentation
Edwards' syndrome: most common trisomy resulting in live birth after Down syndrome, o neurological findings
cause: trisomy 18, nondisjunction. Presentation: mental retardation, unique appearance,  muscle weakness
rocker-bottom feet, micrognathia, low-set ears, clenched hands with overlapping fingers,  loss of deep tendon reflexes
prominent occiput, congenital heart disease (VSD), death < 1 y/o  loss of vibratory sensation and proprioception
 clumsy gait with falls, nystagmus
Patau's syndrome: cause: trisomy 13: nondisjunction. Presentation: mental retardation, o other findings
unique appearance: microphthalmia, microcephaly, cleft lip/palate, holoprosencephaly,  pes cavus
polydactyly, VSD, cystic kidneys, death < 1 y/o  diabetes mellitus
 hypertrophic cardiomyopathy
Cri-du-chat syndrome: cause: microdeletion of short arm of chromosome 5. huntington’s disease: inheritance pattern: autosomal dominant
Presentation: high-pitched crying/mewing (origin of name: French for cry-of-the-cat), Repeat expansion: CAG repeat on chromosome 4
microcephaly, moderate to severe mental retardation, epicanthal folds, VSD Presentation: neurologic findings:
• caudate atrophy with enlarged ventricles on head CT
Williams syndrome: cause: microdeletion of long arm of chromosome 7 (region lost • decreased GABA and Ach
includes elastin gene); presentation: distinctive "elfin" facies, mental retardation, • increased dopamine
hypercalcemia (↑ sensitivity to Vitamin D), unique behaviors, well-developed verbal • NMDA mediated excitotoxicity
skills, extreme friendliness with strangers, musical talent, supravalvular aortic stenosis • movement disorder
• aggression/pyschosis
22q11 microdeletions: cause: microdeletion at chromosome 22q11, abnormal • depression
embryological development of 3rd and 4th pharyngeal pouch. Variable pressenation: • dementia
CATCH-22 disease: cleft palate, abnormal facies, T-cell deficiency (due to thymic aplasia), occurs typically a young patient (age 20 - 50)
cardiac abnormalities, hypocalcemia (due to parathyroid aplasia, results in tetany). mnemonic: HUNT 4 an animal, put it in a CAGe

Specific presentation of 22q11 microdeletions: myotonic dystrophy type 1:


DiGeorge syndrome: defects in thymus, parathyroid, heart  Inheritance pattern
Velocardiofacial syndrome: defects in palate, face, heart; no abnormalities thymus, o autosomal dominant
parathyroid  Repeat expansion
Trinucleotide repeat expansion disease: o CTG repeat on DMPK gene
Fragile X syndrome:
 results in
 Inheritance pattern  abnormal expression of myotonin protein kinase
o X-linked dominant
 Presentation
o myotonia gene). Presentation: severe cognitive disability, frequent seizures, ataxia, speech
o muscle wasting impairment, hyperactivity, inappropriate laughter ("happy puppet"); affects both male
o cataracts and females
o testicular atrophy Population genetics: Forces responsible for genetic variation:
o arrhythmia Mutation: de novo mutation rates constant among populations; intrinsic error rate in
o frontal balding DNA polymerase
o classically "can't release a doorknob/handshake" founder effect: if one member of a small community carries a triat, as the population
epigenetics: expands there will be a higher frequency of that trait in the new community than there is
 Changes in gene expression caused by mechanisms other than changes in actual in the general population. Ex.) Pennsylvania Amish and Ellis-van Creveld syndrome
DNA sequence
 Examples genetic drift: a dramatic change in allele frequency based on chance; small populations
o X-inactivation are more vulnerable to genetic drift
o imprinting
o histone modification natural selection: ↑ in allelic frequency that ↑ species fitness, ↓ in allelic frequency that ↓
x- inactivation: species fitness. some genes ↑ species fitness as heterozygote but ↓ species fitness as a
homozygote ex.) sickle cell trait lowers malarial infections, while sickle cell anemia is
 Overview
o normalizes the genetic amount of males and females (lyonization) detrimental
o inactivates # of X chromosomes - 1 in a Barr body
bottleneck: Even when fitness is equal for all phenotypes, a population bottleneck can
 triploid X will have 2 Barr bodies
result in disrupted allelic frequencies or loss of a genotype all together by chance
 Mechanism
o mediated by XIST gene gene flow: transfer of alleles from one population to another
o inactivation through methylation
o occurs at blastocyst stage in female embryos Hardy-Weinberg equilibrium: states that genotype and allele frequencies remain
o X copy chosen for inactivation is random constant through generations: disease prevalence equation: p2+ 2pq + q2 = 1, where p =
 after choosing every subsequent cell will have the same X frequency of allele A, where q = frequency of allele B, p2 = frequency of homozygous
copy inactivated individuals for allele A, q2 = frequency of homozygous individuals for allele B, 2pq =
 Clinical relevance frequency of heterozygotes
o mosaicism
 non-homogenous X inactivation requirements for validity: large population, random mating
 some cells express paternal X and some cells express
maternal X the genotypic frequencies of the population will remain stable from generation to
imprinting: describes differences in transcriptional activity based on whether the generation, assumptions: no mutation, no selection for any of the genotypes at the locus,
chromosome is of maternal or paternal origin, at a single locus: 1 allele is active, 1 allele no migration
is inactive: creates a hemizygous state
Mechanism: inactive ("imprinted") allele is methylated during gametogenesis, creates other notes: prevalence of an X-linked recessive disease in males = q; prevalence of an X-
transcriptional inactivity; is maternal/paternal specific (gene at one locus always linked recessive disease in females = q2, possible to assume in most cases that p = 1 as
methylated on a specific copy), all cells of an individual have same imprinting level. the wild-type allele is approximately 1
During gametogenesis of the individual the methylation state is erased, reset to be either
maternal or paternal depending on the sex gene mapping: linkage analysis:
Principles
Clinical importance:  linkage
Prader-Willi syndrome: cause: deletion of normally active paternal allele on 15q; o the rate of cosegregation of two syntenic alleles (alleles appearing on
remember: Prader = paternal deletion. Presentation: mental retardation, hyperphagia → the same chromosome) is inversely proportional to the distance
obesity, hypogonadism, neonatal hypotonia, behavior problems; affects both male and between the alleles
females  i.e. genes that are physically close on a chromosome will be
inherited together more often
Angelman's syndrome: cause: disruption of the maternally expressed and paternally  genes that are physically far apart will be inherited together
imprinted gene UBE3A, which encodes an E3 ubiquitin ligase, typically results from less often
deletion of the normally active maternal allele on chromosome 15q, same region of the
 linkage group
genome as Prader-Willi syndrome deletion but opposite chromosome (not the same
o set of genes at different loci on the same chromosome that tend to act Genetic identification: forensic/paternity testing; use of variable number tandem repeats
as a single set of genes in meiosis rather than undergoing independent (VNTRs) or short tandem repeats (STRs): unique copies of non-coding regions of DNA
assortment between individuals, since they exist on both chromosomes, individuals have two copies
 an exception is if crossing over occurs at each locus, 1 paternal and 1 maternal; can only prove with certainty that the sample
 linkage analysis DOES NOT belong to the test subject, cannot prove with 100% certainty that DNA
o using a pedigree to determine the location of an allele based on the belongs to individual of interest because there is a small chance that someone shares the
rate of cosegregation with other alleles of known location (genetic same VNTR or STR
markers)
o can be used to track disease-causing mutation Direct mutation: if DNA region is known, PCR can amplify that region for sequencing
 linkage analysis can localize a gene responsible for a trait
when the sequence is unknown RT-PCR:
 centi-Morgan (cM): Function: used to measure the amount of RNA present in a sample
o a unit used to measure genetic linkage Process: reverse transcriptase is added to solution containing RNA, dNTPs, primers for
o a length of DNA over which the frequency of homologous specific sequence of interest, and heat stable DNA polymerase: RNA is converted to DNA
recombination is 1% and DNA sequence of interest is amplified, the amount of amplified PCR product after a
 1% chance that a marker on a chromosome will become set number of PCR cycles is directly proportional to the concentration of RNA in the
separated from a second marker on the same chromosome original sample
due to crossing over
Clinical use: HIV viral load: measures transcriptional activity of the virus by detecting the
 two loci that are 1 centimorgan apart will experience
amount of RNA present, gives a more detailed picture of the infection and treatment
recombination between these two loci in 1% of meiosis.
results
 parental gamete
o the portion of chromosome looks the same as that of one of the Blotting:
parents Function: probe for specific substance in solution:
o no recombination Southern blot - used to analyze DNA; normally used to examine the presence of a
 nonparental gamete particular DNA sequence, remember: Southern Dixieland
o homologous recombination has occurred in the region of interest Northern blot - used to analyze RNA; normally used to examine gene expression
 linkage disequilibrium Western blot (immunoblot) - used to analyze protein
o when alleles occur together more often than can be accounted for by
chance Process: run a gel electrophoresis to separate the components of the solution, bands are
 indication that 2 alleles are physically close together transferred ("blotted") to a filter/membrane, radiolabeled or fluorescently labeled probe
Biochemical lab technique: PCR: polymerase chain reaction: is incubated with the membrane: Southern blot - 32P-DNA, Northern blot - 32P-DNA,
Function: can amplify a selected region of DNA Western blot - enzyme-linked or 131I antibody. Specific probe binds with high specificity
Process: solution prepared containing: DNA primers specific for selected DNA region, to DNA/RNA segment or protein of interest, membrane is visualized under conditions to
DNA sample of interest, heat stable DNA polymerase, deoxyribonucleotides; illuminate the probe
denaturation of dsDNA by heating, annealing of DNA primer specific for region of
interest and slowly cooling the solution, replication of DNA at the primer by heat stable Clinical use:
DNA polymerase, repetition of the process several times, gel electrophoresis used to restriction length polymorphism (RFLP): can be used for genetic testing; based on the
separate the various components of the solution principle that individuals have unique/heritable variations in RFLPs, loss or gain of a
restriction site changes the fragment lengths after digestion by an endonuclease,
Clinical use: in all uses PCR functions to amplify the amount of DNA present in a sample; variation provided in part by variable number tandem repeats (VNTRs). Southern blot
high specificity bacterial/viral infection testing: used to visualize results ex.) testing for sickle-cell anemia: wild type (wt) gene contains a
restriction site in the gene fragment, sickle cell (sc) gene lacks this restriction site, the
HIV: first step is ELISA (high sensitivity), PCR is used to determine viral load, test diagnostic results following a Southern blot: wt homozygote would have only medium
examines the amount of viral DNA integrated into host cell DNA. Advantages over ELISA: and small fragments, heterozygote would have large, medium and small fragments, sc
PCR becomes positive earlier in disease course (Positive ELISA result is dependent on homozygote would have only large fragments
antibody formation), PCR does not require that the patient have a competent immune
system (ELISA requires the host to make antibodies), important specific cases when PCR gene expression profiling: measure whether a particular region of DNA is being
should always be used: (a newborn whose mother is HIV+ will have antibodies even if expressed, Northern blot used to visualize results ex.) testing for fragile X syndrome: ↓
not infected, so ELISA does not work) when earliest possible detection is required expression of FMR1 gene, trinucleotide repeat disorder
microarrays:  because mRNA has had the non-coding introns spliced out
Function: can probe for thousands of different mRNAs simultaneously it can express genes from the cloned host
Process: a chip has thousands of DNA sequences robotically attached on a grid, a solution  bacteria do not contain the enzymes to splice RNA
of degraded mRNA is added to the chip, thousands of probes are added, a computer  requires reverse transcriptase to convert mRNA into DNA
analyzes the degree of binding to various regions of the chip for the library
 similar mechanism of accessing the library as the genomic
Clinical use: analyze cancer cell gene expression; can determine probability of becoming library; however the probe is an 125I-antibody against the
malignant based on population comparison studies, can be used to guide therapy protein of interest
o gene therapy
Cloning:  allows introduction of a normal copy of a gene into a cell that
 Function was defective in that gene
o transfer production of a protein from one organism to another  uses a delivery vector to introduce the gene
o synthesize large quantities of a plasmid or its product  usually modified viruses
 Process  viral genome is replaced with plasmid
o insertion of a DNA segment of interest into a vector  virus inserts the gene into the host as part
 vector is usually a plasmid of its normal life cycle
 first: digestion of vector with restriction  problems
endonuclease  viruses can indiscriminately
 second: sealing of the segment into the vector insert copies of gene into
with DNA ligase undesirable regions
o insertion of the vector into a bacteria via transformation  oncogenesis
o selection of the bacteria via a mechanism to distinguish which cells  cells must be dividing for viral
received the vector and those that did not integration
 usually via an antibiotic resistance gene  adenovirus vectors do
 Applications not require active
o recombinant proteins division, but do not
 produced by introduction of a plasmid containing the actually insert into the
protein product of interest into bacteria genome
 bacterial colony grown  can be performed
 some plasmids contain regulatory sequences which  ex vivo
can turn on/off expression of the plasmid  cells removed from the body, modified,
 bacteria lysed and produced protein is purified and transplanted back into the body
 ex.) recombinant insulin, factor VIII, bacterial factors for  in vivo
vaccination  cells modified in the body without
o genome sequencing/libraries removal
 contains the entire genome of an organism spread between  has been used to treat SCID caused by deficiency in the IL-
bacterial colonies receptor γ chain
 the genome of an organism is degraded with a restriction biochemical model systems:
endonuclease at specific palindromic sites  Transgenic mice
 individual pieces are inserted into a bacterial colony and o construction
cloned individually  germline modification
 to access the library for amplification of a particular segment  cloned gene (transgene) introduced into a fertilized
a blot is taken of the library and radiolabeled segment of mouse ovum
interest is hybridized  constitutive
 the labeled colony (in which hybridization took place) can be  non-specific insertion into genome
harvested and grown  conditional
o cDNA libraries  specific (targeted) insertion into genome
 "expression libraries"  the altered phenotype with the added gene is observed
 contain all the expressed genes of an organism in a similar through the entire lifespan
fashion as a genomic library
 including embryonic development karyotyping:
 number of gene copies is variable Function: examine chromosomal structure
 altered phenotype passed to offspring as it is a germline Process: mitotic cells in metaphase selected from actively dividing cell population (white
mutation cells, bone marrow, placenta, amnion; when chromosomes are most condensed),
o function chromosomes are stained with or without G-banding (G-banding involves partial
 studying dominant disorders digestion with trypsin, leaves alternating dark and white bands), staining with Giemsa or
 due to the fact that the transgene will coexist with other stains which have affinity for DNA
normal gene product
 Knockout mice spectral karyotyping: 5 different fluorescent probes are added, special digital image
o construction processing colors each chromosomes differently; chromosomes are ordered, numbered
 similar to transgenic mice except instead of adding a foreign based on unique properties of all 23 chromosomes
gene, a normal gene is removed
Clinical use: can analyze whether a gross chromosomal abnormality exists ex.) trisomy,
 also a germline modification
monosomy; used commonly for prenatal diagnosis via chorionic or amniotic sampling.
o function
spectral karyotyping can also diagnose translocations, inversions, or deletions of specific
 useful for studying loss-of-function mutations in human
chromosomal segments if the change is large enough
diseases
 Chimeric mice FISH: fluorescence in situ hybridization:
o construction
 Function
 a genetic mix between two distinct cell lines
o detect the presence/location of a specific region of DNA in the genome
 fusion of two early embryogenic lines in embryogenesis
 two lines recognized as self due to absence of immune  Process
system
o a probe for a desired DNA sequence is created with a fluorescent tag
o function o probe is added to DNA sample and color is detected
 can reproduce to create offspring of various types in  Clinical use
subsequent generations o detect microdeletions, translocations, and aneuploidies
 non-transgene homozygous  similar role to karyotyping however the DNA does not have
 transgene homozygous to be condensed during mitosis to be visualized
 transgene heterozygous  ex.) a probe for chromosome 7 would only have one signal if
the patient had a monosomy as opposed to 2 in a normal
 Cre-lox system
sample
o construction
 can also be used to diagnose Prader-Willi syndrome (15q)
 a system that allows turning on/off of gene of interest with deletion
an antibiotic-controlled promoter
DNA sequencing:
o function
 to study a gene whose deletion is lethal to embryos  Function
o determining sequence for a desired DNA sample
 allows the embryo to develop to a certain stage and
can then turn off a certain gene  Process (Sanger/direct method)
 can also study the timing of when a gene is important in o DNA sample put in 4 different reaction mixtures
development  each contains all 4 deoxynucleotides, 1 different
dideoxyneucleotide (ddNTP), and a DNA polymerase
 RNAi
o dideoxynucleotides stop DNA polymerization at each base
o construction
 generates DNA chains of various lengths that encompass the
 dsRNA is synthesized complementary to the mRNA sequence
entire original sequence
of interest
o solutions are separated by gel electrophoresis
 transfected into cells
o sequence can be deduced by looking at each electrophoresis row and
 dsRNA separates and binds the complementary mRNA reading which ddNTP was added to that tube column
 dsRNA (where one strand is the host mRNA) is seen as a
viral product and degraded
 Clinical use
o function o most commonly used when no specific set of mutations is a common
cause of disease
 ↓ expression of a particular gene

You might also like